Are you seeking one-on-one college counseling and/or essay support? Limited spots are now available. Click here to learn more.

How to Write the AP Lang Synthesis Essay with Example

September 5, 2023

AP Lang synthesis essay, AP Language

If you’re highly interested in learning more about writing analysis, then chances are you enrolled in AP Lang. Essentially, AP Lang is an advanced course for high schoolers that combines interest and knowledge in English with critical thinking. In the class, students learn how to analyze and synthesize a variety of texts to construct well-reasoned arguments. If you take AP Lang, then you can opt to take the AP test at the conclusion of the school year. On the exam, students write the AP Lang synthesis essay to demonstrate their learned abilities. In this article, we’ll look at what the AP Lang synthesis essay requires and show an example to provide better understanding of what to expect on the exam.

AP Lang Exam Basics

The AP Lang exam is separated into two sections. In the first section, students have one hour to answer a series of 45 multiple-choice questions. Here, about half of the questions are based on passages students read. The other half are focused on the best revision techniques. Essentially, the answers for the latter 20-22 questions are geared toward revising mock essays.

In this article, however, we’ll focus mainly on the second part of the exam: the AP Lang synthesis essay.

In this second section, students have two hours and 15 minutes to write three essays of their own design. The three open-ended questions in this section are intended to be free-response and allow for a variety of approaches. Each question is intended to allow up to 40 minutes to complete.

For the AP Lang synthesis essay, students are presented with a scenario of the College Board’s design. The scenario will provide its own thesis statement. Usually, scenarios relate to real-world problems like environmental concerns, media, or government policies.

For each scenario, students are provided with 6-7 outside sources. These sources could be in the form of an image, visual graph, or written paragraph. For written paragraphs, the sources are usually no more than 500 words.

Students are then expected to incorporate at least 3-4 of these outside sources into their essay response. The outside sources are intended to be used as supporting evidence for the student’s chosen stance or argument. Students are able to either agree with or disagree with the thesis presented in the original scenario.

AP Lang Exam – Scoring

In the second part of the AP Lang exam, students can earn a possible 6 points on each essay. 1 point is earned for the development of a thesis. Up to 4 points can be earned for evidence and commentary. The final 1 point is earned for sophistication of thought.

AP Lang Exam – Takeaways

Ultimately, the goal of the AP Lang synthesis essay is not whether the student is “right” or “wrong” in their argument. The key is that students are able to reasonably and clearly support their argument using the provided sources as evidence .

The College Board looks for your ability to identify relationships between texts , form a coherent argument , and interpret external sources .

Synthesis Essay AP Lang Examples

If you’re not sure how the questions will look on the AP Lang synthesis essay section, we’ll provide an example. After the example, we’ll break down the strengths and weaknesses of the response. That way, you’ll have a better idea of what the College Board is looking for.

Additionally, the College Board has released previous AP Lang synthesis essay examples you can review. They even have essay questions as recent as 2022 . For further support, a scoring commentary and comments from the Chief Reader are also available to view. Additionally, there are other examples you can view from earlier years .

Note: A good strategy to study for the synthesis essay AP Lang exam is to review your rhetorical devices and literary devices . Understanding how these devices function can be essential in constructing a cohesive essay.

Synthesis Essay AP Lang Examples – Sample Question

Below is a sample question from the AP Lang synthesis essay and a response to the prompt. This question was taken directly from a 2022 exam . However, the response to the question will be originally crafted for the purpose of this newsletter. As well, all supporting evidence will be originally created and does not correspond to any previous test.

The Question

Since the early 2000s, the United States government and a number of corporations have sponsored initiatives to improve education in the STEM disciplines: science, technology, engineering, and mathematics. The emphasis on STEM subjects in elementary, secondary, and higher education reflects concerns that United States students are less proficient in these areas than are students in other countries. Additionally, there is a belief that mastery in STEM fields is now essential in order to join a highly technical and specialized workforce. However, not everyone is convinced that a STEM-focused curriculum is necessary and/or effective.

In your response you should do the following:

  • Respond to the prompt with a thesis that presents a defensible
  • Select and use evidence from at least three of the provided sources to support your line of Indicate clearly the sources used through direct quotation, paraphrase, or summary. Sources may be cited as Source A, Source B, etc., or by using the description in parentheses.
  • Explain how the evidence supports your line of
  • Use appropriate grammar and punctuation in communicating your

How to Approach the Question

Maybe your first thought upon seeing this block of text is to feel overwhelmed. But don’t panic. There are effective ways to approach the question so you will be more prepared in your response.

It’s a good strategy to first isolate the thesis . What is the main idea of the text, and what is its argument?

Try it out. Reread the prompt and see if you can identify what the statement is asking you to develop an opinion on.

Think you’ve got it? In this example, we will be focusing on whether or not a STEM-focused curriculum in K-12 education is necessary and/or effective. In short, we will be arguing either for (highlighting the benefits) or against (highlighting the pitfalls) a STEM-focused curriculum.

How do we know what this statement is asking us?

Well, the statement provides a lot of background information. For example, we receive a definition of what STEM stands for. As well, we know that since 2000, there has been a greater initiative for STEM-focused classes.

When you read the prompt for the first time, it’s a great strategy to learn how to differentiate between background and contextual information from the heart of the argument .

A good way to learn how to isolate the argument is to look for transition words. Usually, these appear near the end of the question. Words like “however” and “yet” are signals that the statement is offering a differing opinion. Typically, the statement will tell you which two positions it’s offering for argument. These opinions are usually signaled by contrasting transition words.

So, now that we know what the question is asking us, what is the best way to respond?

Synthesis Essay AP Lang Examples – Sample Answer

The following is an essay response I crafted to the above question. After reading the sample, I will break down what it does well and what areas can be improved.

A STEM-focused curriculum is not as essential to providing a meaningful K-12 education. Because the majority of high school students are not proficient in STEM-focused classes, prioritizing these classes causes harm to student’s mental health and academic performance.

As seen in Source A, 60% of high school seniors in the Midwest only scored a C average in math and science-based classes (Langston). This statistic suggests that the majority of students do not resonate with STEM classes and therefore perform poorly. Earning a low score in any class does not bode well for students’ mental health.

When looking at the primary argument in Source C, it’s clear that most high schoolers prefer creative outlets to fact-based research (Kohler). Allowing students the opportunity to be more creative and initiate conversations about coursework lets students be more active in their learning. When students can discuss the nuance in their opinions, more personal growth happens. These conversations are not always easy to have in STEM-focused classes.

As well, when looking back to Source A, it’s clear that high school students in the Midwest earned higher grades, on average, in their English and art classes (Langston). This figure suggests that students perform better in these classes because they relate more to the source material. When relating to what they learn, they perform better in class.

In conclusion, STEM-focused curriculum is not as essential in K-12 education because most high school students do not relate to their STEM classes. When students do not earn satisfactory grades in these classes, it negatively affects their future college applications and job prospects.

Synthesis Essay AP Lang Examples – Answer Breakdown

So, what does this essay response get right, and where can it be improved? Let’s start with what the response does well.

First, the response establishes its thesis right away. Usually, it’s a good idea to clearly state your argument within the first paragraph. Not only is this a good practice because a reader can easily identify your stance, but also you can refer to your thesis as you write to make you stay on track.

With your thesis, it’s also a good idea to include one to two supporting sentences with the reasons why the thesis is concluded . Like in this example, I wrote that STEM-focused classes should not be prioritized because they can negatively affect both mental health and academic performance.

Another positive aspect of this response is that it is sure to not only reference but also cites its sources . It’s important that the reader understand where your information is coming from. That way, the readers can ensure you are interpreting the sources correctly.

AP Lang Synthesis Essay (Continued)

However, when rereading the instructions, it’s clear that this response fails the basic requirement of referring to at least three sources. Always make sure to reread the instructions to ensure you meet the standard requirements for incorporating source material.

Further, this AP Lang synthesis essay does not fully support its arguments . Ideas are simply stated and are not expanded upon.

For example, I mentioned a few times that earning low grades in STEM classes leads to negative mental health for high school students. However, there is no source referenced that either confirms or denies this claim. Therefore, there is no sufficient evidence to support my argument. It relies purely on inference.

Additionally, this AP Lang synthesis essay does not arrive at a sufficient level of sophistication of thought . Basically, sophistication of thought means avoiding broad generalizations and vague claims. The more specific you can be, the better your argument will sound.

Synthesis Essay AP Lang – In Conclusion

In the end, it’s always helpful to read the prompt thoroughly before writing. As well, making notes while you read could be a good strategy to pinpoint main ideas both in the prompt and the sources. That way, you can reread the material quickly. Similarly, sketching an outline may also be helpful. In addition, you should always carefully read the instructions to ensure all guidelines are followed.

As long as you avoid broad generalizations and use enough supporting evidence for your claim, you will be on the right path!

  • High School Success

' src=

Meghan Dairaghi

With a BA in English and an MFA in Creative Writing, Meghan has served as a writing tutor at the University of Missouri St. Louis and Maryville University. Additionally, Meghan has held editorial roles at River Styx and Boulevard, and was a prose reader at Farside Review . Most recently, her work has been featured in Belle Ombre , Flypaper Lit , and Mag 20/20 , among others, and she was nominated for the Mary Troy Prize in Fiction. 

  • 2-Year Colleges
  • Application Strategies
  • Best Colleges by Major
  • Best Colleges by State
  • Big Picture
  • Career & Personality Assessment
  • College Essay
  • College Search/Knowledge
  • College Success
  • Costs & Financial Aid
  • Data Visualizations
  • Dental School Admissions
  • Extracurricular Activities
  • Graduate School Admissions
  • High Schools
  • Homeschool Resources
  • Law School Admissions
  • Medical School Admissions
  • Navigating the Admissions Process
  • Online Learning
  • Outdoor Adventure
  • Private High School Spotlight
  • Research Programs
  • Summer Program Spotlight
  • Summer Programs
  • Teacher Tools
  • Test Prep Provider Spotlight

College Transitions Sidebar Block Image

“Innovative and invaluable…use this book as your college lifeline.”

— Lynn O'Shaughnessy

Nationally Recognized College Expert

College Planning in Your Inbox

Join our information-packed monthly newsletter.

I am a... Student Student Parent Counselor Educator Other First Name Last Name Email Address Zip Code Area of Interest Business Computer Science Engineering Fine/Performing Arts Humanities Mathematics STEM Pre-Med Psychology Social Studies/Sciences Submit

how to write essay for ap lang

Crafting an Impressive Argumentative Essay for AP Lang

how to write essay for ap lang

Writing an argumentative essay for AP Language and Composition requires a strategic approach to effectively convey your perspective. Here's a guide to crafting an impressive argumentative essay:

1. Understand the Prompt:

   - Tip: Carefully read and analyze the prompt. Identify the key elements, including the task you are asked to perform and any specific requirements or constraints.

2. Develop a Clear Thesis Statement:

   - Tip: Formulate a concise and focused thesis statement that clearly states your argument or position. This statement should guide the reader on what to expect in your essay.

3. Identify Your Audience:

   - Tip: Consider your target audience and tailor your language and arguments accordingly. Understanding your audience helps you establish a connection and effectively convey your message.

4. Organize Your Essay Effectively:

   - Tip: Structure your essay with a clear introduction, body paragraphs, and a conclusion. Ensure a logical flow of ideas, with each paragraph contributing to the overall argument.

5. Provide Context and Background:

   - Tip: Begin with a brief introduction that provides context for your argument. Clearly state the issue at hand and offer background information to help readers understand the significance of your argument.

6. Present a Strong Claim:

   - Tip: Clearly articulate your main claim or argument. This should be a debatable statement that forms the core of your essay. Avoid vague or overly broad claims.

7. Support Your Argument with Evidence:

   - Tip: Use relevant and compelling evidence to support your claims. This can include facts, statistics, examples, anecdotes, or quotations. Ensure that your evidence is credible and directly contributes to your argument.

8. Address Counterarguments:

   - Tip: Acknowledge potential counterarguments and address them in your essay. This demonstrates a nuanced understanding of the issue and strengthens your overall argument.

9. Use Persuasive Language:

    - Tip: Choose language that is persuasive and impactful. Utilize rhetorical devices, vivid imagery, and compelling language to engage your readers and convey the urgency or importance of your argument.

10. Craft Well-Reasoned Body Paragraphs:

    - Tip: Dedicate separate paragraphs to different aspects of your argument. Each paragraph should have a clear topic sentence, supporting evidence, and a concise explanation of how the evidence relates to your thesis.

11. Ensure Cohesive Transitions:

    - Tip: Use transitional phrases and sentences to ensure a smooth transition between paragraphs. This enhances the coherence of your essay and guides readers through your line of reasoning.

12. Conclude Effectively:

    - Tip: Summarize your main points in the conclusion and restate the significance of your argument. Avoid introducing new information in the conclusion. End with a strong closing statement that leaves a lasting impression.

13. Revise and Edit:

    - Tip: Set aside time for revision and editing. Review your essay for clarity, coherence, and grammatical accuracy. Ensure that your argument is compelling and well-supported.

14. Seek Feedback:

    - Tip: If possible, seek feedback from peers, teachers, or mentors. External perspectives can provide valuable insights and help you identify areas for improvement.

15. Stay Focused on the Argument:

    - Tip: Throughout the essay, stay focused on your central argument. Avoid unnecessary tangents or diverging from the main point. Every part of your essay should contribute to strengthening your overall argument.

Crafting an impressive argumentative essay for AP Language and Composition involves a combination of persuasive writing techniques, solid evidence, and effective organization. By following these tips, you can create a compelling essay that showcases your ability to engage in sophisticated argumentation and rhetorical analysis.

You Might Also Like

how to write essay for ap lang

The Ultimate Guide to College Interviews

College interviews, although nerve wrecking, can be the best chance for you to impress the admissions officer. We give you tips on how to ace your interview.

how to write essay for ap lang

How to Ace Your Common App

Why common application play important role in college admission procedure? Get details about common application and how to create common app account - Read more

how to write essay for ap lang

Scholarship Application Process

Filling out college scholarship applications is indeed a time-consuming task that needs much effort and patience. This guide will help you through the entire scholarship application process

AP Guru has been helping students since 2010 gain admissions to their dream universities by helping them in their college admissions and SAT and ACT Prep

Free Resources

Calculate for all schools

Your chance of acceptance, your chancing factors, extracurriculars, writing a synthesis essay for ap lang.

I have to write a synthesis essay this week for my AP Lang class, but I'm not 100% sure what exactly that entails. Can someone explain what the process is like and maybe share some tips on how to write a strong synthesis essay?

Sure, a synthesis essay is a type of essay that requires you to use multiple sources to create an argument. In an AP Lang synthesis essay, you'll typically be provided with the sources and will need to analyze them, identify the main ideas, and then connect those ideas to your central argument or thesis.

Here's a step-by-step breakdown of how to write a strong synthesis essay for AP Lang:

1. Understand the prompt : Carefully read the prompt and make sure you understand what's being asked. Identify the main issue or topic you'll be addressing.

2. Develop a thesis statement : Your thesis statement should make a clear and specific claim about the topic. It should be an arguable point that you'll support using evidence from the sources provided.

3. Read and analyze the sources : Read through each source, taking notes on key ideas, evidence, and arguments related to your thesis. Think about how each source contributes to your argument and look for connections between sources.

4. Outline your essay : Create an outline that organizes your ideas and evidence into a logical sequence. Typically, a synthesis essay will have an introduction, body paragraphs, and conclusion. Each body paragraph should focus on a specific point related to your thesis and include evidence from multiple sources to support the point.

5. Write the introduction : Begin with a hook to engage the reader and introduce the topic. Then, give some brief background information about the issue before presenting your thesis statement.

6. Write the body paragraphs : In each body paragraph, start with a topic sentence that introduces the main point of the paragraph. Then, provide evidence and analysis from the sources to support the point, making sure to cite your sources according to the citation style required by your instructor. Try to incorporate evidence from multiple sources and emphasize the connections between them.

7. Address counterarguments : It's important to address any opposing views or counterarguments to show that you've considered different perspectives. You can either refute counterarguments or concede some validity while still maintaining your overall argument.

8. Write the conclusion : Summarize your main points and restate your thesis statement in a new way. You can also suggest further implications of your argument or propose a call to action. Avoid introducing new information in the conclusion.

Some additional tips for writing a strong synthesis essay:

- Make sure your analysis connects effectively to your thesis statement.

- Clearly explain how the evidence from the sources supports your argument.

- Use appropriate transitions to guide your reader through the essay and help them understand the connections between ideas.

- Edit and revise your essay for clarity, grammar, and punctuation.

Good luck with your AP Lang synthesis essay!

About CollegeVine’s Expert FAQ

CollegeVine’s Q&A seeks to offer informed perspectives on commonly asked admissions questions. Every answer is refined and validated by our team of admissions experts to ensure it resonates with trusted knowledge in the field.

logo-type-white

AP® English Language

How to get a 6 on the argument frq in ap® english language.

  • The Albert Team
  • Last Updated On: March 1, 2022

How to Get a 6 on the Argument FRQ in AP® English Language

What We Review

Introduction: How to Get a 6 on the Argument FRQ in AP® English Language

Wondering how to get a 6 on the argumentative essay in AP® English Language? 

To score an 5 on the AP® English Argument FRQ question, the CollegeBoard scoring guidelines outline that students need to write an essay that effectively argues a position, uses appropriate and convincing evidence, and showcases a wide range of the elements of writing. Essays that score a 6 do all of that and, additionally, demonstrate sophistication in their argument.

An essay that does all of that is an incredibly well-constructed essay. Such an essay needs a solid framework and excellent support. To do this, it is important to have a clear idea of what you are being asked, to not waffle, to spend time and care with your thesis and outline, and to support every claim you make.

We know the best way to write an AP® English FRQ that does everything right is to understand what you are going to see on the AP® English Language test. Read on to prepare yourself for exam day and earn that 6!

What to Expect from the AP® English Language Argument Free Response Questions

The AP® English argument FRQ is the most straightforward of the AP® English FRQs because it is the most similar to the essays you’re already used to writing. It’s exciting to have free reign and make your own argument, unrestrained from rhetorical analysis devices or documents. But, like most AP® writing, it also can be a little overwhelming.

There’s nothing to read and analyze to provide evidence or help you form an argument. Whether you’re feeling excited or overwhelmed by the AP® writing argument FRQ, consider the rhetorical situation. Be strategic about forming your thesis, craft a strong, chronological argument, and utilize good, supportive evidence to earn a better overall essay response.

Determine the question.

The first question to ask yourself is what am I being asked to do ? This may seem obvious, but it’s surprising how tricky it can be to figure out. Look for keywords and phrases that will answer that question.

Here’s an example from the 2019 AP® English Language argumentative essay.

What to Expect from the AP® English Language Argument Free Response Questions - Determine the Question

Though there are just two short paragraphs, there is a lot of room for confusion here. In this case, “Then, write a well-developed essay in which you explain your judgment.” is the key sentence you are looking for. In 2019, AP® English Language test takers were asked to select a concept, place, role, etc. that they believed was “overrated,” and explain why.

If you cannot determine what the question is, go back and reread the prompt. Focus on the last few sentences, as that’s where you’ll usually find it.

Knowing the question you are answering is the most important part of AP® writing. You will not be able to answer the question effectively if you aren’t certain what the question is. Pick out a specific sentence or two to determine the question, and thereby ensure that you aren’t just writing an essay that responds to the general sense of the argument essay prompts

Pick an opinion and stick to it.

The next step is both simple and difficult. Identify your own opinion on the subject.

But remember — the AP® argumentative essay exam format is designed to test how well you can craft an argument. Questions like the 2019 question seem so daunting, because claiming anything to be “overrated” is such a broad topic. It is a bigger question than students are used to encountering on an AP® test.

But, always remember, there is no right or wrong answer for this AP® English FRQ. And whatever argument you choose will not come back later in the exam or in your final grade in the class. This is not to say that you shouldn’t believe in what you are writing. Only that you should remember that both sides are arguable, pick one, and stick to it. Don’t waffle.

Below we break down two sample student answers from this same 2019 prompt. 

What to Expect from the AP® English Language Argument Free Response Questions - PIck an opinion negative example

In this AP® Lang argument essay example, the student jumps from describing places, to people, to outfits. The prompts asked for only one example and the student gives three.  By doing this, it shows they were not only unable to grasp what the prompt was asking, but that they couldn’t stick to their opinion.  Instead of deeply strengthening one choice, the student gives vague, half-reasons for too many choices. When writing your FRQs, choose just one concept and stick to it.

The following example demonstrates a strong student response:

What to Expect from the AP® English Language Argument Free Response Questions - Pick an opinion strong example

This student picks one clear concept, capitalism, and clearly outlines their support for it.  They write with clear language that opens the door for the deeper analysis coming later in the essay.

Like this student, choose just one clear argument to delve into when writing your FRQ.

Craft a thesis statement.

The thesis statement should be both simple and elegant. Students often find it one of the more difficult writing skills to master, but we’re here to help. Just remember that it should encompass your entire essay in just one sentence.  So, for the 2019 argument FRQ :

Good thesis: While capitalism undeniably has its upsides, it has many downsides that are rarely recognized. When considering the downsides, capitalism is clearly overrated as it commodifies humanity and uplifts a minority at the expense of the majority.

This thesis breaks down a) that the author clearly states his claim that capitalism is overrated, b) that the author will support that claim with examples on how it commodifies humanity and how it hurts the majority in favor of the minority.

Good thesis: While the Electoral College was created in the name of equality for smaller states, it is ultimately overrated because it undercuts the popular vote, it is an archaic practice that is unsuitable for the modern era.

This thesis claims the Electoral College is overrated by claiming it doesn’t do what it was created to do in the first place- support equality.  It also introduces two supporting examples for the rest of the essay- it undercuts the popular vote and it doesn’t work in the modern era.

Not a good thesis: Kicking a ball in a net and scoring, is not as important as saving lives. Soccer to me would be considered overrated.

This thesis doesn’t give clear direction for the rest of the essay.  The author claims soccer is overrated, but doesn’t tell us why. The example that “it’s not as important as saving lives” is unrelated and also not touched on again later in the essay. This thesis isn’t specific and doesn’t give you a clear idea of what the author will be saying next.

Not a good thesis: The term “overrated” has been used in conversation to diminish the value of roles. In unusual circumstances the term “overrated” should be applied to the idea of freedom in regards to social change, but overall it should not be applied in regards to global devastation and cruel treatment.

This thesis does not directly answer the question.  Is the author arguing that freedom is overrated? They also claim that the term overrated doesn’t apply to global devastation and cruel treatment. This second claim is both unrelated to the first and doesn’t work to answer the initial prompt.

Looking at these four examples, can you see the difference between a strong and weak thesis?

After you’ve determined your thesis, use it as a jumping point to sketch a quick outline. Then, follow your outline, bringing in your own concrete examples and evidence. Doing so will improve your AP® writing.

Return to the Table of Contents

Craft a chronological argument.

A good argument builds as you move through the essay. It does not simply repeat the same points. Instead, the different points of the argument build off one another and work together to advance the author’s point.

Let’s look at the 2018 AP® English argument FRQ for an example.

What to Expect from the AP® English Language Argument Free Response Questions - Craft a chronological argument

In this case, students are being asked to argue a position on the value of choosing the unknown. 

All students are likely to have their own definitions of what “choosing the unknown” might mean. You first want to consider what this phrase means to you, and how it applies to the real world.  Could it mean breaking out of your comfort zone in daily routines, or could it mean going to theater school to follow your dreams?  There’s no wrong answers, but try to pin down one. Consider Lindbergh’s quote the prompt gives you, and how shock, disappointment, and enrichment play into choosing the unknown.

Once you’ve nailed down your definition, you can begin to form your arguments. A chronological argument builds off itself. So, in this question’s case, an outline would look something like this:

  • Choosing the unknown is necessary for the development of the human race.
  • Scientific advancements cannot be made without testing the boundaries of the unknown.
  • Cultural and artistic growth can only occur through exploring the unknown.

First, a student must define what choosing the unknown means, and what makes it difficult. Next the student argues for the value of choosing the unknown, in that the human race could never develop without it.  Finally, the student will argue for the invaluable scientific and cultural/artistic advances made throughout history by breaking known boundaries.

When you sketch your outline, quickly ask yourself if the outline would make just as much sense if you rearranged it. If the answer is no, start writing your essay. If the answer is yes, try to structure your argument so that your points build off one another.

Support your claims.

All arguments need evidence. This is the proof you need to support your thesis. And in the case of the AP® English argument FRQ, the evidence all comes from you. What exactly that evidence is will vary from question to question and from student to student. But make sure that every point you make is supported by evidence.

Here’s some good news — you already know quite a bit about effective evidence from what you have learned in AP® English about rhetorical devices. Your main purpose in this essay is to persuade. What have you learned in class about effective ways to persuade? What rhetorical devices can you utilize? Try to pick the best devices to support your argument that you can.

Here are some examples of supportive and non-supportive evidence that students could use to support their claims.

What to Expect from the AP® English Language Argument Free Response Questions - Support your claims

The 2017 AP® English language argument FRQ asked students to argue a position if the most essential skill is artifice. The example student answers given below are from here .

Supportive evidence:   “Throughout history, rulers have utilized countless different methods of achieving power, however none have been so successful as mastering the art of lying.

In his advice to future rulers, Niccolo Machiavelli encouraged them to lie and maintain the illusion of sympathy to the common struggles in order to retain power. He asserts that it is imperative for a ruler to appear caring and sympathetic even if he has no objective but power.

Machiavelli argues that to be sincere and honest is akin to being vulnerable. A ruler must be skilled in the art of deception if he is not to fall prey to usurpers. Thus, it is essential that he appear humble and morally upright to his constituents as he is to appear idealistic, despite his nature being identical to his citizens.”

In this paragraph, the student chooses to discuss the role of artifice in politics. The student claims that mastering lying is essential to achieving political power. The student uses Machiavelli’s leadership and beliefs as specific examples to support this, by analyzing and connecting each point back to his/her claim.

Non-supportive evidence: “Another example would be actors on red carpets or at interviews they sound generous and relatable, but in reality they could be selfish people who don’t care about anyone. To the public they act charming, honest, and sincere. They do this so they can get famous and rich. They do this so they will never get ignored.”

In this paragraph, the student chooses to discuss the role of artifice in the culture of entertainment and celebrities. However, the student does not utilize supportive evidence to do so. The paragraph is full of claims about how actors lie, but does not provide a concrete example to anchor the claims. The student provides a lot of very vague generalizations, but no clear evidence or examples of specific celebrities and how they used artifice to succeed.

There is so much variance in prompts and students’ prior knowledge; it’s impossible to provide a checklist of what makes evidence supportive. But a good trick to decide if you’ve supported your claims well enough is to talk to yourself. No really, it’s a good idea.

Picture yourself discussing your essay with someone. Imagine that this person disagrees with everything that you say. Every time you make a claim, like that it’s important to be polite in an email, your imaginary person shakes their head and tells you no. How would you try to convince them? What examples would you use? Make sure that for each opinion you put forward; you have provided an answer to someone who would disagree with you.

The evidence is an important part of your essay. If your outline and your argument are a framework, your evidence is the brick and mortar. A house without brick and mortar won’t fall, but it won’t be a very nice house to inhabit. Tie every claim you make to a piece of evidence to ensure the best essay possible.

Wrapping Things Up: Scoring a 6 on the Argument FRQ for AP® English Language

The AP® English argument FRQ varies quite a bit. But it is ultimately about how well you can put forth an argument. So, don’t be afraid to spend some time crafting that argument. We’ve covered a lot in this article- here are the main points to remember:

  • Determine the question. Figure out what the prompt is asking you to do.
  • Pick an opinion and stick to it. Choose one side of the argument and one clear claim to support all the way through.
  • Craft a thesis statement. Your thesis should be clear, concise, and introduce the content of your essay.
  • Craft a chronological argument. Make an argument that builds on its prior points.
  • Support your claims. Support yourself with concrete, specific evidence and examples. 

But most of all, have fun. This essay is the one you should be looking forward to, where you have the freest rein. Enjoy it and earn yourself a 6.

Do the examples shown make sense to you? Can you picture yourself moving through the AP® writing argument FRQ with ease now?

Interested in a school license?​

8 thoughts on “how to get a 6 on the argument frq in ap® english language”.

Thank you for explaining this so eloquently. Excellent post, I will keep this handy and refer to it often from now on. It’s so educative. Great post!

Sure, glad it helped.

I’m an AP® Language teacher and the title of your article caught my eye because the essays aren’t scored on a 0-9 scale anymore. The max score for an essay now is a 6. Essays are now scored in 3 categories: Thesis: 0 or 1 point Evidence and commentary: 0-4 points Sophistication: 0 or 1 point I just wanted to let you know! I saw this was last updated in 2020 and just thought it should reflect the current AP® exam.

Thank you for the heads up! This is an older blog post that must have had something else updated to it this year. We’ve gone ahead and revised the post.

Hi, my AP® Language teacher emphasized on a counterargument at the end of the supporting paragraphs. Could you elaborate on it? Also, how exactly do we get the sophistication point?

Hi Stephanie, thanks for reaching out! Making a solid counter-argument is definitely one way to make sure that you earn the Sophistication point. We recommend having a look at our AP® English Language Review Guide for 2021 for more tips! The College Board’s Free-Response Question and Scoring Information Archive also provides authentic examples of student writing — many of which successfully make counterarguments and rebuttals to earn the Sophistication point.

Hi can I get a 6?

Hi Roy, we certainly believe that earning a 6 on your FRQs is possible with practice and dedication! I’d recommend having a look at our AP® English Language Review Guide for tips and tricks, and you can also browse our AP® English Language and Composition Resource Page and Free Response practice questions for targeted practice.

Comments are closed.

Popular Posts

AP® Physics I score calculator

AP® Score Calculators

Simulate how different MCQ and FRQ scores translate into AP® scores

how to write essay for ap lang

AP® Review Guides

The ultimate review guides for AP® subjects to help you plan and structure your prep.

how to write essay for ap lang

Core Subject Review Guides

Review the most important topics in Physics and Algebra 1 .

how to write essay for ap lang

SAT® Score Calculator

See how scores on each section impacts your overall SAT® score

how to write essay for ap lang

ACT® Score Calculator

See how scores on each section impacts your overall ACT® score

how to write essay for ap lang

Grammar Review Hub

Comprehensive review of grammar skills

how to write essay for ap lang

AP® Posters

Download updated posters summarizing the main topics and structure for each AP® exam.

how to write essay for ap lang

How to Write the AP Lang Synthesis Essay

how to write essay for ap lang

AP Lang test is the logical conclusion to the introductory college English composition course. And its most important (and often difficult) part is the AP Lang synthesis essay. Despite it being the very basic layer of your future composition skills, it’s a very complicated challenge to approach unprepared. Besides, it's details may change year to year. So let’s have a look with our coursework writing services team at what your AP Lang exam 2022 might look like.

What is AP Lang?

AP Lang is a relatively lengthy test. There are several AP rubrics that a student must be well-versed in to hope to pass it. The first section includes reading and writing, while the second is slightly more freeform and includes three different types of essays.

Among those three, the most interesting and, coincidentally, oftentimes the hardest to deal with is the AP Lang synthesis essay rubric. Today will focus on it specifically to make sure you know exactly what you’re going to be facing during your test.

What Is a Synthesis Essay AP Lang?

At its core, the AP Lang synthesis essay is a pretty straightforward part of the AP Lang test. It might look pretty similar to the reading section of the exam. However, simply finding the right information isn’t enough. When writing a synthesis essay, you should not only gather the data but also distill it into your personal opinion.

This fine line may seem difficult to spot, but it is there. And it’s that small difference that can make or break your exam run. So try to follow the steps one by one and not lose focus. Writing a good synthesis essay is as easy as following the rules. If you feel this task is too difficult for you, you can leave us your ' write an essay for me ' request and we will do it for you.

AP Lang Synthesis Essay Outline

Looking through AP Lang essay examples, you might notice that the overall structure doesn’t really differ too much from your standard essay outline. You have your introduction, your body, and your conclusion. But the important thing to note is where your arguments are supposed to come from.

You’re not supposed to just go off on a rant. The task requires you to base your supporting evidence on at least three sources. And you will have to ensure your essay has solid roots. Here’s what a basic AP Lang exam synthesis essay outline should look like:

  • Introduction

Provide sufficient context for the topic you are about to cover. You can do a quick overview of prevailing opinions you have grasped while browsing through your source materials.

Write a short and compelling thesis statement. This will be your ground zero for the rest of the essay. So make sure it reflects your opinion. What is a thesis statement you can read in our special article.

  • Body Paragraphs

Dedicate at least one paragraph to every source you’re using. Start with presenting the evidence you have gathered from that source and go on to explain how it formed your opinion on the topic and why it should be considered.

Quickly go through your line of reasoning and reinforce what you have already covered. Finish up with restating your thesis as you’re supposed to logically arrive at it after all the evidence you have presented. That’s how you write a conclusion properly.

Different Forms and Types of Synthesis Essay: Explanatory vs. Argumentative Synthesis Essays

When it comes to writing a synthesis essay AP Lang, there are several types of essays you should consider. The most common ones are the AP Lang argument essay and explanatory essay. The clues as to how each of them should look are hidden within their names but let’s go over them to clear any confusion.

An explanatory essay’s goal is to go over a certain topic, discuss it in detail, and ultimately show a high level of understanding of the said topic. You don’t necessarily have to get into a heated argument with the reader trying to convince them of something. All you need to do is create an impartial overview.

On the other hand, an argumentative essay has to do with personal opinions. And while there is a time and a place for bias, it still has to be as impartial and factual as possible. When proving your point, try not to devolve into emotional arguments but stick to logic and cold truths. This will make your argument way more solid.

Synthesis Essay Structure

In the general case, you don’t really need to look for a synthesis essay AP Lang example to get a solid grasp on how its structure should look like. You can safely fall back on your high school essay writing knowledge, and you’ll be mostly safe.

What you should pay attention to is your writing style and content. A synthesis essay is identified less by its structure and more by the way you form and present your arguments to the reader. It’s when you get a specific essay type (like an argumentative essay) that you should pay attention to slight changes in format.

Argument Essay Structure

The best way to understand argumentative essay structure is to study any well-written AP Lang argument essay example. Standard AP Lang essays have very distinctive features that are very easy to spot and emulate. They follow a very rigid form and employ specific rhetorical devices that you’ll be able to pick up after you analyze them once or twice quickly.

How Many Paragraphs Should an AP Lang Synthesis Essay Be?

The number of paragraphs in an AP Lang synthesis essay can indeed make a difference. Your arguments should be concise and pointed. Spreading them out throughout many paragraphs may seem like a good idea to fill in the space. But it’s actually detrimental to your final score. You can get a basic understanding of what your score is going to be using an AP Lang score calculator.

The same goes for too few paragraphs. Don’t even try to squeeze your entire line of thought into a single body paragraph. Generally, the minimum number of sources you should address is three. Any less, and you are getting a lower score. So try to keep it somewhere in the middle. Three to five body paragraphs is an optimal number. Don’t forget to add an intro and a conclusion to it and you’re all set. A well-written essay has a clear and easily identifiable structure.

How to Write AP Lang Synthesis Essay: Guide

How to Write the AP Lang Synthesis Essay

In order to write a decent essay, all you have to do is follow these simple steps. Performing a rhetorical analysis essay example, AP Lang won’t give you insight into how it was built from the ground up. But looking at this list might.

Step 1. Read the Prompt

It may sound like a no-brainer. But it’s actually more important than you can imagine. Don’t skip right past this step. It’s very easy to misunderstand the task under stress. And if you do slip up in the beginning - the entirety of your work after that is wasted.

Step 2. Analyze the Sources Carefully

The same goes for your sources. Take your time reading them. Try to spot every smallest detail, as even a single one can help you better incorporate your evidence into the body of your essay. You can begin outlining the general points of your essay in your head at this point.

Step 3. Come Up with a Strong Thesis Statement

Your thesis statement is the baseline of your writing. Make it short and clear. Try not to overthink it too much.

Step 4. Fill in Your Essay Outline

Start filling out your outline step by step. You don’t have to go from top to bottom. If you feel like you’re struggling - skip to the next part and return to the problem paragraph later. The use of rhetorical devices AP Lang is also pretty important. So once you flesh out your essay a bit, spend some time trying to come up with the perfect wording.

Step 5. Finalize

The first finished version of your essay is a draft. Don’t be hasty to turn it in. Read over it a couple of times. Make sure everything is in order. You can switch some of the parts around or rewrite some sections if you have the time. Ideally, at this stage you should have enough time to eliminate all grammatical errors that may still be present in your essay. Polish it to perfection.

Useful Tips

Here are some useful tips that might make the writing process a bit easier for you:

  • Use either APA or Chicago style to cite your sources
  • Have a schedule to understand how much time you have for each section
  • Leave as much time as you can for editing and proofreading
  • You can never over study the source material. Spend as much time as you can reading into it
  • Don’t linger on the surface of your essay subject. Dive in and show your complex understanding of the material
  • Avoid using private life anecdotes to support your case unless the essay type specifically allows it. These don’t make for a convincing argument.
  • Use as many supporting arguments as you can but make sure they are actually solid and relevant to your thesis
  • Check with your thesis from time to time. The entirety of your text should align with it

Need help with academic deadlines?

Falling back on your deadlines? Use our term paper writing services to relieve you while you get back on your feet.

AP Lang Essay Prompts

Here are some interesting prompts. Some of them could be found in the previous iterations of the test; you may have spotted them in some of the AP Lang essay examples. Others are there to help you practice for the AP Lang exam 2022.

  • The John F. Kennedy Presidential Library and Museum, dedicated in 1979, was founded in memory of the president and contained archives pertaining to his administration. On June 24, 1985, then President Ronald Reagan joined members of the Kennedy family at a fundraising event to help the Kennedy Library Foundation create an endowment to fund and support the presidential library. The following is an excerpt from the speech Reagan gave at that event. Read the passage carefully. Write an essay that analyzes the rhetorical choices Reagan makes to achieve his purpose of paying tribute to John F. Kennedy.
  • On August 29, 2009, then-President Barack Obama delivered a eulogy at the funeral of Senator Ted Kennedy in Boston, Massachusetts. Kennedy served in the United States Senate from 1962 until his death. Obama served with him in the Senate from 2005 until Obama was elected president in 2008. The following is an excerpt from Obama’s speech. Read the passage carefully. Write an essay that analyzes the rhetorical choices Obama makes to achieve his purpose of praising and memorializing Kennedy.
  • On April 9, 1964, Claudia “Lady Bird” Johnson, who was at the time the First Lady of the United States, gave the following speech at the first-anniversary luncheon of the Eleanor Roosevelt Memorial Foundation. The foundation is a nonprofit division of the Franklin D. Roosevelt Presidential Library dedicated to the works of former First Lady Eleanor Roosevelt, who passed away in 1962. Read the passage carefully. Write an essay that analyzes the rhetorical choices Johnson makes to achieve her purpose of paying tribute to Eleanor Roosevelt.

In your response, you should do the following:

• Respond to the prompt with a thesis that analyzes the writer’s rhetorical choices.

• Select and use evidence to support your line of reasoning.

• Explain how the evidence supports your line of reasoning.

• Demonstrate an understanding of the rhetorical situation.

• Use appropriate grammar and punctuation in communicating your argument.

AP Lang Essay Example

Here is a decent if a bit shortened, AP Lang rhetorical analysis essay example you can use for reference.

Literature to Prepare for AP Lang

How to Write the AP Lang Synthesis Essay

And here is a list of some great AP Lang books that will help you prepare for the exam. Not all of them are immediately useful, but most will help you enhance your writing and analytical abilities to get a better score in the end.

  • The Odyssey
  • Don Quixote
  • A Midsummer Night's Dream
  • Pride and Prejudice
  • Wuthering Heights
  • Oliver Twist
  • Crime and Punishment
  • Adventures of Huckleberry Finn

If you have thoughts of "who could do my paper for me," do not forget that you can contact us. Or, if you have a finished paper and you need to make edits to it, leave us a ' rewrite my essay ' request and we will do it as soon as possible.

Daniel Parker

Daniel Parker

is a seasoned educational writer focusing on scholarship guidance, research papers, and various forms of academic essays including reflective and narrative essays. His expertise also extends to detailed case studies. A scholar with a background in English Literature and Education, Daniel’s work on EssayPro blog aims to support students in achieving academic excellence and securing scholarships. His hobbies include reading classic literature and participating in academic forums.

how to write essay for ap lang

is an expert in nursing and healthcare, with a strong background in history, law, and literature. Holding advanced degrees in nursing and public health, his analytical approach and comprehensive knowledge help students navigate complex topics. On EssayPro blog, Adam provides insightful articles on everything from historical analysis to the intricacies of healthcare policies. In his downtime, he enjoys historical documentaries and volunteering at local clinics.

informative essay

AP English Language and Composition: Sample Rhetorical Analysis and Synthesis Questions

April 9, 2024.

AP English Language Sample Rhetorical Analysis and Synthesis Questions

The Rhetorical Analysis and Synthesis Essays are two of the three essays you’ll need to write as part of the AP English Language and Composition Exam . Read on for a sample of each, as well as tips for how to answer them. 

AP English Language and Composition: Sample Rhetorical Analysis Question

Read the following passage published back in 1967 by The New York Times. Then write an essay in which you analyze how the structure of the passage and the use of language help convey the writer’s views.

Sample Question Instructions:

  • Respond to the prompt with a thesis that may establish a line of reasoning.
  • Select and use evidence to develop and support the line of reasoning.
  • Explain the relationship between the evidence and the thesis.
  • Demonstrate an understanding of the rhetorical situation.
  • Use appropriate grammar and punctuation in communicating the argument.

Americans and Western Europeans, in their sensitivity to lingering problems around them, tend to make science and progress their scapegoats. There is a belief that progress has precipitated widespread unhappiness, anxieties, and other social and emotional problems. Science is viewed as a cold mechanical discipline having nothing to do with human warmth and the human spirit. 

But to many of us from the nonscientific East, science does not have such repugnant associations. We are not afraid of it, nor are we disappointed by it. We know all too painfully that our social and emotional problems festered long before the age of technology. To us, science is warm and reassuring. It promises hope. It is helping us at long last gain some control over our persecutory environments, alleviating age-old problems—not only physical but also, and especially, problems of the spirit.

Shiraz, for example, a city in southern Iran, has long been renowned for its rose gardens and nightingales; its poets, Sadi and Hafiz; and its mystical, ascetic philosophy, Sufism. Much poetry has been written in glorification of the spiritual attributes of this oasis city. And to be sure, Shiraz is a green, picturesque town, with a quaint bazaar and refreshing gardens. But in this “romantic” city thousands of emotionally disturbed and mentally retarded men, women, and children were, until recently, kept in chains in stifling prison cells and lunatic asylums. 

Every now and again, some were dragged, screaming and pleading, to a courtyard and flogged for not behaving “normally.” But for the most part, they were made to sit against damp walls, their hands and feet locked in chains, and thus immobilized, without even a modicum of affection from their helpless families and friends, they sat for weeks and months and years—often all their lives. Pictures of these wretched men, women, and children can still be seen in this “city of poetry,” this “city with a spiritual way of life.” 

It was only recently that a wealthy young Shirazi who, against the admonitions of his family, had studied psychology at the University of Tehran and foreign universities, returned to Shiraz and after considerable struggle with city officials succeeded in opening a psychiatric clinic, the first in those regions. After still more struggle, he arranged to have the emotionally disturbed and the mentally retarded transferred from prison to their homes, to hospitals, and to his clinic, where he and his staff now attend them. 

They are fortunate. All over Asia and other backward areas, emotionally disturbed men and women are still incarcerated in these medieval dungeons called lunatic asylums. The cruel rejection and punishment are intended to teach them a lesson or help exorcise evil spirits. 

The West, still bogged down in its ridiculous romanticism, would like to believe that emotional disturbances, dope addiction, delinquency are all modern problems brought on by technological progress, and that backward societies are too spiritual and beautiful to need the ministrations of science. But while the West can perhaps afford to think this way, the people of backward lands cannot. . . . 

. . .The obstacles are awesome, the inertia too entrenched, the people’s suffering too anguished, their impatience too eruptive. Moreover, the total cultural reorganizations such as Asia and Africa are undergoing inevitably engender their own temporary dislocations and confusions. But their goals, the direction, remain constant. We are on the move, however awkwardly at first, to a saner, better world.

How to Answer the AP English Language and Composition Rhetorical Analysis Question

Go back to the original question, which asks you to analyze two features of the passage: (1) its structure, or organization, and (2) its language. The first aspect is fairly specific. As you read the passage, you need to observe what the author discusses first, second, third, and so on. Your essay should explain not only the order of ideas but the reasons the author may have chosen that order. 

The second part of the question is more general. It invites you to analyze the use of language, which may include the author’s choice of words (diction), syntax (word order), figures of speech, use of evidence (such as statistics or logical reasoning), sentence structure, rhythm, sound, tone, or just about any other characteristics of style and rhetoric you choose. 

Although the question directs you to write about two different aspects of the passage, the essay itself should be unified. That is, a good essay should not consist of, say, two disparate paragraphs, one exclusively devoted to structure and another to language. Rather, the essay should include material that shows the interrelationship of structure and language in the passage and how those elements contribute to the meaning and effect of the passage. This might be covered in a separate paragraph, or it could be woven into the overall fabric of the essay. 

Before you begin to write, read the passage at least twice: once for an overview and once as you write your analysis. You may notice early on that the opening paragraph contains generalizations about Westerners’ concepts of science and progress. Then the author contrasts the Western view of science and progress with the Eastern view. Immediately, you see that the author, by using the first-person pronoun (as in “many of us”) is speaking from the perspective of an Easterner. Consequently, his discussion of Eastern views is apt to come across as more well-informed, more authoritative, perhaps more personal. 

To support his position, the author gives an extended example—the city of Shiraz—to illustrate just how different the East is from the West. The description and vivid images of Shiraz memorably convey the idea that the “spiritual way of life” has a side to it that many Westerners don’t know about. This is the heart of the passage. The use of quotation marks around “romantic” and “city of poetry” is meant to point out the discrepancy between the idealized and real versions of Shiraz. 

Nearing the end, the author reiterates his initial contrast between West and East, with emphasis on the East. The last paragraph offers a generalized statement about conditions in Asia and Africa, reminding the reader of the contrast made at the very beginning of the passage. Tying the end to the beginning of the passage creates a sense of unity—a desirable feature in any piece of writing.

AP English Language and Composition: Sample Argument Question

The following paragraph is adapted from Mirror for Man, a book written by anthropologist Clyde Kluckhorn in the middle of the twentieth century. Read the passage carefully. Then, write an essay that examines the extent to which the author’s characterization of the United States holds true today. Use appropriate evidence to support your argument. 

Sample Question Instructions: 

  • Respond to the prompt with a thesis that may establish a line of reasoning. 
  • Select and use evidence to develop and support the line of reasoning. 
  • Explain the relationship between the evidence and the thesis. 
  • Demonstrate an understanding of the rhetorical situation. 

Technology is valued as the very basis of the capitalistic system. Possession of gadgets is esteemed as a mark of success to the extent that persons are judged not by the integrity of their characters or by the originality of their minds but by what they seem to be—so far as can be measured by their wealth or by the variety and material goods which they display. “Success” is measured by their investments, homes, and lifestyles— not by their number of mistresses as in some cultures.

How to Answer the AP English Language and Composition Argument Question

Whether you agree, disagree, or have mixed views on the content of the passage, your job is to write a convincing argument that expresses your opinion. Initially, the word argument may suggest conflict or confrontation. But rest assured that your essay need not be combative. Rather, make it a calmly-reasoned explanation of your opinion on a debatable subject. Your goal is to persuade the reader that your opinion, supported by examples, facts, and other appropriate evidence, is correct. 

If you have strong feelings about the topic, of course you should state them in your essay. But express them in calm, rational language. Be mindful that the essay should not be an emotional rant for or against the issue. 

Consider first whether you agree with Kluckhorn’s definition of “success.” Is it, as Kluckhorn asserts, measured by income and material possessions? Or do you think that a more accurate standard of success in today’s America should be determined by less tangible criteria—things such as happiness or self-respect? Or do you stand somewhere in between those two extremes? 

The actual position you take on the issue is less crucial than your ability to support it fully by drawing from your knowledge, background, experience, or observation. Regardless of your position, be sure to include more than one example. An argument that relies on a single example, however compelling, will fall flat. 

In the prompt, Kluckhorn’s notion of success seems to refer broadly to American society. Resist responding in kind. That is, a short essay shouldn’t focus on the whole of society but only on an identifiable segment—perhaps college-educated professionals or urban, blue- collar Americans. The point is that a narrowly focused essay on a limited topic will always turn out better than one that tries to cover too much ground in just a few paragraphs.

AP Biology Resources

  • About the AP Biology Exam
  • Top AP Biology Exam Strategies
  • Top 5 Study Topics and Tips for the AP Biology Exam
  • AP Biology Short Free-Response Questions
  • AP Biology Long Free-Response Questions

AP Psychology Resources

  • What’s Tested on the AP Psychology Exam?
  • Top 5 Study Tips for the AP Psychology Exam
  • AP Psychology Key Terms
  • Top AP Psychology Exam Multiple-Choice Question Tips
  • Top AP Psychology Exam Free Response Questions Tips
  • AP Psychology Sample Free Response Question

AP English Language and Composition Resources

  • What’s Tested on the AP English Language and Composition Exam?
  • Top 5 Tips for the AP English Language and Composition Exam
  • Top Reading Techniques for the AP English Language and Composition Exam
  • How to Answer the AP English Language and Composition Essay Questions 
  • AP English Language and Composition Exam Sample Essay Questions
  • AP English Language and Composition Exam Multiple-Choice Questions

AP Human Geography Resources

  • What’s Tested On the AP Human Geography Exam?
  • AP Human Geography FAQs
  • AP Human Geography Question Types and Strategies
  • Top 5 Study Tips for the AP Human Geography Exam

FOLLOW ALONG ON SOCIAL

Scores for 2024 AP Exams are available now. Sign in and get your reports . 

Synthesis Essay Materials

The two synthesis essay questions below are examples of the question type that has been one of the three free-response questions on the AP English Language and Composition Exam as of the May 2007 exam. The synthesis question asks students to synthesize information from a variety of sources to inform their own discussion of a topic. Students are given a 15-minute reading period to accommodate the additional reading required for the question.

Below is a sample synthesis essay question, sample scoring guidelines, comments from the Chief Reader about the sample student essays, seven sample student responses, and scoring commentary for each sample.

Approximately 300 AP English Language and Composition students from eight schools in New York, Maine, Texas, Tennessee, Washington, Florida, and New Mexico wrote responses to this synthesis topic. Students from these schools were given a 15-minute reading period followed by a 40-minute writing period in which to complete the sample synthesis assignment.

  • AP English Language and Composition Synthesis Essay #1 (.pdf/84KB)
  • Sample Scoring Guidelines (.pdf/43KB)
  • Comments from the Chief Reader (.pdf/47KB)
  • Sample Student Responses (.pdf/3.65MB)
  • Scoring Commentary (.pdf/50KB)

An additional sample synthesis essay question is provided here.

  • AP English Language and Composition Synthesis Essay #2 (.pdf/338KB)

PrepScholar

Choose Your Test

  • Search Blogs By Category
  • College Admissions
  • AP and IB Exams
  • GPA and Coursework

Expert Guide to the AP Language and Composition Exam

author image

Advanced Placement (AP)

article-71342_640.jpg

With the 2023 AP English Language and Composition exam happening on Tuesday, May 9, it's time to make sure that you're familiar with all aspects of the exam. In this article, I'll give a brief overview of the test, do a deeper dive on each of the sections, discuss how the exam is scored, offer some strategies for studying, and finally wrap up with some essential exam day tips.

Exam Overview

The AP Language and Composition exam tests your rhetorical and composition skills. Essentially, how do authors construct effective arguments in their writing? What tools do they use? How can you use those tools to craft effective writing yourself? That is the essence of rhetorical analysis.

The exam has two parts: the first section is an hour-long, 45 question multiple-choice section. It includes five sets of questions, each based on a passage or passages. In this section, there will be 23-25 rhetorical analysis questions which test your rhetorical skills. There will also be 20-22 writing questions which require you to consider revisions to the texts you're shown.

The second section is free response. It starts with a 15-minute reading period, and then you'll have 120 minutes to write three analytical essays:

  • One essay where you synthesize several provided texts to create an argument
  • One essay where you analyze a nonfiction passage for its rhetorical construction
  • One essay where you create an original argument in response to a prompt.

You will have about 40 minutes to write each essay, but no one will prompt you to move from essay to essay—you can structure the 120 minutes as you wish.

In the next sections I'll go over each section of the exam more closely—first multiple choice, and then free response.

The AP English Language and Composition Multiple-Choice

The multiple-choice section tests you on two main areas. The first is how well you can read and understand nonfiction passages for their use of rhetorical devices and tools. The second is how well you can "think like a writer" and make revisions to texts in composition questions.

You will be presented with five passages, about which you will receive a small amount of orienting information, e.g. "This passage is excerpted from a collection of essays on boating" or "This passage is excerpted from an essay written in 19th-century Haiti." Each passage will be followed by a set of questions.

There are, in general, eight question types you can expect to encounter on the multiple-choice section of the exam. I've taken my examples from the sample questions in the " Course and Exam Description ."

eight-1316133_640.jpg

Magic eight-ball says there are eight types of multiple-choice questions!

Type 1: Reading Comprehension

These questions are focused on verifying that you understood what a certain part of the passage was saying on a concrete, literal level. You can identify these questions from phrases like "according to" "refers," etc. The best way to succeed on these questions is to go back and re-read the part of the passage referred to very carefully.

Comprehension.png

Type 2: Implication

These questions take reading comprehension one step further—they are primarily focused on what the author is implying without directly coming out and saying it. These questions will have a correct answer, though, based on evidence from the passage. Which interpretation offered in the answers does the passage most support? You can identify questions like these from words like "best supported," ‘"implies," "suggests," "inferred," and so on.

implies.png

Type 3: Overall Passage and Author Questions

These questions ask about overall elements of the passage or the author, such as the author's attitude on the issue discussed, the purpose of the passage, the passage's overarching style, the audience for the passage, and so on.

You can identify these questions because they won't refer back to a specific moment in the text. For these questions, you'll need to think of the passage from a "bird's-eye view" and consider what all of the small details together are combining to say.

3overall_passage.png

Type 4: Relationships Between Parts of the Text

Some questions will ask you to describe the relationship between two parts of the text, whether they are paragraphs or specific lines. You can identify these because they will usually explicitly ask about the relationship between two identified parts of the text, although sometimes they will instead ask about a relationship implicitly, by saying something like "compared to the rest of the passage."

4relationship.png

Type 5: Interpretation of Imagery/Figurative Language

These questions will ask you about the deeper meaning or implication of figurative language or imagery that is used in the text. Essentially, why did the author choose to use this simile or this metaphor? What is s/he trying to accomplish?

You can generally identify questions like this because the question will specifically reference a moment of figurative language in the text. However, it might not be immediately apparent that the phrase being referenced is figurative, so you may need to go back and look at it in the passage to be sure of what kind of question you are facing.

5imagery.png

Type 6: Purpose of Part of the Text

Still other questions will ask you to identify what purpose a particular part of the text serves in the author's larger argument. What is the author trying to accomplish with the particular moment in the text identified in the question?

You can identify these questions because they will generally explicitly ask what purpose a certain part of the text serves. You may also see words or phrases like "serves to" or "function."

6purpose_of_part.png

Type 7: Rhetorical Strategy

These questions will ask you to identify a rhetorical strategy used by the author. They will often specifically use the phrase "rhetorical strategy," although sometimes you will be able to identify them instead through the answer choices, which offer different rhetorical strategies as possibilities.

7rhetorical_strategy.png

Type 8: Composition

This is the newest question type, first seen in the 2019/2020 school year. For these questions, the student will need to act as though they are the writer and think through different choices writers need to make when writing or revising text.

These questions can involve changing the order of sentences or paragraphs, adding or omitting information to strengthen an argument or improve clarity, making changes to draw reader attention, and other composition-based choices.

body_composition

Some very important stylish effects going on here.

The AP English Language and Composition Free Response

The free response section has a 15-minute reading period. After that time, you will have 120 minutes to write three essays that address three distinct tasks.

Because the first essay involves reading sources, it is suggested that you use the entire 15-minute reading period to read the sources and plan the first essay. However, you may want to glance at the other questions during the reading period so that ideas can percolate in the back of your mind as you work on the first essay.

Essay One: Synthesis

For this essay, you will be briefly oriented on an issue and then given anywhere from six to seven sources that provide various perspectives and information on the issue. You will then need to write an argumentative essay with support from the documents.

If this sounds a lot like a DBQ , as on the history AP exams, that's because it is! However, this essay is much more argumentative in nature—your goal is to persuade, not merely interpret the documents.

Example (documents not included, see 2022 free response questions ):

body-AP-Literature-synthesis

Essay Two: Rhetorical Analysis

In the second essay, you'll be presented with an excerpt from a nonfiction piece that advances an argument and asked to write an essay analyzing the rhetorical strategies used to construct the passage's argument. You will also be given some orienting information—where the passage was excerpted from, who wrote it, its approximate date, where it was published (if at all), and to whom it was directed.

Example (excerpt not included, see 2022 free response questions ):

body-AP-literature-Question-2

Essay Three: Argument

In the third essay, you will be presented with an issue and asked to write a persuasive essay taking a position on the issue. You will need to support your position with evidence from your "reading, experience, and observations."

body-AP-lit-Question-3

This doesn't look like a very well-constructed argument.

How The AP Language and Composition Exam Is Scored

The multiple-choice section of the exam is worth 45% of your score, and the free-response section is worth the other 55%. So each of the three free-response essays is worth about 18% of your score.

As on other APs, your raw score will be converted to a scaled score of 1-5. This exam has a relatively low 5 rate. Only 10% of test takers received a 5 in 2022 , although 56% of students received a score of 3 or higher.

In terms of how the raw score is obtained, the multiple-choice section is similar to other AP multiple-choice sections: you receive a point for every question you answer correctly, and there is no penalty for guessing.

The grading rubrics for the free-response questions were revamped in 2019. They are scored using analytic rubrics instead of holistic rubrics. For each free-response question, you will be given a score from 0-6. The rubrics assess three major areas:

#1: Thesis (0 to 1 points): Is there a thesis, and does it properly respond to the prompt?

#2: Evidence and Commentary (0 to 4 points): Does the essay include supporting evidence and analysis that is relevant, specific, well organized, and supports the thesis?

#3: Sophistication (0 to 1 points): Is the essay well-crafted and does it show a sufficiently nuanced understanding of the prompt?

Each scoring rubric broadly assesses these three factors. However, each task is also different in nature, so the rubrics do have some differences. I'll go over each rubric—and what it really means—for you here.

Synthesis Essay Rubrics

0 For any of the following:
1

EVIDENCE AND COMMENTARY

0
1 AND
2 AND
3 AND
4 AND

SOPHISTICATION

0
1 Responses that earn this point may demonstrate sophistication of thought and/or a complex understanding of the rhetorical situation by doing any of the following:

dough-196235_640.jpg

Time to synthesize this dough into some cookies.

Rhetorical Analysis Essay Rubrics

0
1 AND
2 AND
3 AND AND
4 AND AND

magnifying-glass-914922_640.png

Examine your texts closely!

Argumentative Essay Rubrics

playing-puppies-790638_640.jpg

The best kind of frenzy is a puppy frenzy!

AP English Language Prep Tips

Unlike its cousin, the AP English Literature and Composition exam, the AP Language and Composition exam (and course) have very little to do with fiction or poetry. So some students used to more traditional English classes may be somewhat at a loss as to what to do to prepare.

Luckily for you, I have a whole slate of preparation tips for you!

Read Nonfiction—In a Smart Way

A major thing you can do to prepare for the AP Lang and Comp exam is to read nonfiction— particularly nonfiction that argues a position , whether explicitly (like an op-ed) or implicitly (like many memoirs and personal essays). Read a variety of non-fiction genres and topics, and pay attention to the following:

  • What is the author's argument?
  • What evidence do they use to support their position?
  • What rhetorical techniques and strategies do they use to build their argument?
  • Are they persuasive? What counterarguments can you identify? Do they address them?

Thinking about these questions with all the reading you do will help you hone your rhetorical analysis skills.

Learn Rhetorical Terms and Strategies

Of course, if you're going to be analyzing the nonfiction works you read for their rhetorical techniques and strategies, you need to know what those are! You should learn a robust stable of rhetorical terms from your teacher, but here's my guide to the most important AP Language and Composition terms .

  • We've compiled a list of 20 rhetorical devices you should know.
  • A heroic individual from Riverside schools in Ohio uploaded this aggressively comprehensive list of rhetorical terms with examples. It's 27 pages long, and you definitely shouldn't expect to know all of these for the exam, but it's a useful resource for learning some new terms.
  • Another great resource for learning about rhetorical analysis and how rhetorical devices are actually used is the YouTube Channel Teach Argument , which has videos rhetorically analyzing everything from Taylor Swift music videos to Super Bowl commercials. It's a fun way to think about rhetorical devices and get familiar with argumentative structures.
  • Finally, a great book—which you might already use in your class—is " They Say, I Say. " This book provides an overview of rhetoric specifically for academic purposes, which will serve you well for AP preparation and beyond.

You also need to practice argumentative and persuasive writing. In particular, you should practice the writing styles that will be tested on the exam: synthesizing your own argument based on multiple outside sources, rhetorically analyzing another piece of writing in-depth, and creating a completely original argument based on your own evidence and experience.

You should be doing lots of writing assignments in your AP class to prepare, but thoughtful, additional writing will help. You don't necessarily need to turn all of the practice writing you do into polished pieces, either—just writing for yourself, while trying to address some of these tasks, will give you a low-pressure way to try out different rhetorical structures and argumentative moves, as well as practicing things like organization and developing your own writing style.

once-upon-a-time-719174_640.jpg

Not the most auspicious start to an argumentative essay.

Practice for the Exam

Finally, you'll need to practice specifically for the exam format. There are sample multiple-choice questions in the " AP Course and Exam Description ," and old free-response questions on the College Board website.

Unfortunately, the College Board hasn't officially released any complete exams from previous years for the AP English Language and Composition exam, but you might be able to find some that teachers have uploaded to school websites and so on by Googling "AP Language complete released exams." I also have a guide to AP Language and Composition practice tests .

Once you're prepped and ready to go, how can you do your best on the test?

Looking for help studying for your AP exam? Our one-on-one online AP tutoring services can help you prepare for your AP exams. Get matched with a top tutor who got a high score on the exam you're studying for!

AP Language and Composition Test Day Tips

Here are four key tips for test-day success.

board-1193334_640.jpg

You are one hundred percent success!

Interact With the Text

When you are reading passages, both on the multiple-choice section and for the first two free-response questions, interact with the text! Mark it up for things that seem important, devices you notice, the author's argument, and anything else that seems important to the rhetorical construction of the text. This will help you engage with the text and make it easier to answer questions or write an essay about the passage.

Think About Every Text's Overarching Purpose and Argument

Similarly, with every passage you read, consider the author's overarching purpose and argument. If you can confidently figure out what the author's primary assertion is, it will be easier to trace how all of the other aspects of the text play into the author's main point.

Plan Your Essays

The single most important thing you can do for yourself on the free-response section of the AP English Language exam is to spend a few minutes planning and outlining your essays before you start to write them.

Unlike on some other exams, where the content is the most important aspect of the essay, on the AP Language Exam, organization, a well-developed argument, and strong evidence are all critical to strong essay scores. An outline will help you with all of these things. You'll be able to make sure each part of your argument is logical, has sufficient evidence, and that your paragraphs are arranged in a way that is clear and flows well.

Anticipate and Address Counterarguments

Another thing you can do to give your free responses an extra boost is to identify counterarguments to your position and address them within your essay. This not only helps shore up your own position, but it's also a fairly sophisticated move in a timed essay that will win you kudos with AP graders.

envelope-392962_640.jpg

Address counterarguments properly or they might get returned to sender!

Key Takeaways

The AP Language and Composition exam tests your rhetorical skills. The exam has two sections.

The first section is an hour-long, 45 question multiple-choice test based on the rhetorical techniques and composition choices.

The second section is a two-hour free-response section (with a 15-minute initial reading period) with three essay questions: one where you must synthesize given sources to make an original argument, one where you must rhetorically analyze a given passage, and one where you must create a wholly original argument about an issue with no outside sources given.

You'll receive one point for every correct answer on the multiple-choice section of the exam, which is worth 45% of your score. The free-response section is worth 55% of your score. For each free-response question, you'll get a score based on a rubric from 0-6. Your total raw score will be converted to a scaled score from 1-5.

Here are some test prep strategies for AP Lang:

#1 : Read nonfiction with an eye for rhetoric #2 : Learn rhetorical strategies and techniques #3 : Practice writing to deploy rhetorical skills #4 : Practice for the exam!

Here are some test-day success tips:

#1 : Interact with each passage you encounter! #2 : Consider every text's overarching purpose and argument. #3 : Keep track of time #4 : Plan your essays #5 : Identify and address counterarguments in your essays.

With all of this knowledge, you're ready to slay the AP English Language and Composition beast!

animal-1299749_640.png

Noble knight, prepare to slay the AP dragon!

What's Next?

Want more AP Lang review? We have a complete collection of released AP Language practice tests , as well as a list of the AP Lang terms you need to know and a guide to the multiple choice section .

Taking the AP Literature exam? Check out our ultimate guide to the AP English Literature test and our list of AP Literature practice tests .

Taking other AP exams? See our Ultimate Guides to AP World History , AP US History , AP Chemistry , AP Biology , AP World History , and AP Human Geography .

Need more AP prep guidance? Check out how to study for AP exams and how to find AP practice tests .

Want to build the best possible college application?   We can help.   PrepScholar Admissions combines world-class admissions counselors with our data-driven, proprietary admissions strategies. We've guided thousands of students to get into their top choice schools, from state colleges to the Ivy League. We know what kinds of students colleges want to admit and are driven to get you admitted to your dream schools. Learn more about PrepScholar Admissions to maximize your chance of getting in:

These recommendations are based solely on our knowledge and experience. If you purchase an item through one of our links, PrepScholar may receive a commission.

Trending Now

How to Get Into Harvard and the Ivy League

How to Get a Perfect 4.0 GPA

How to Write an Amazing College Essay

What Exactly Are Colleges Looking For?

ACT vs. SAT: Which Test Should You Take?

When should you take the SAT or ACT?

Get Your Free

PrepScholar

Find Your Target SAT Score

Free Complete Official SAT Practice Tests

How to Get a Perfect SAT Score, by an Expert Full Scorer

Score 800 on SAT Math

Score 800 on SAT Reading and Writing

How to Improve Your Low SAT Score

Score 600 on SAT Math

Score 600 on SAT Reading and Writing

Find Your Target ACT Score

Complete Official Free ACT Practice Tests

How to Get a Perfect ACT Score, by a 36 Full Scorer

Get a 36 on ACT English

Get a 36 on ACT Math

Get a 36 on ACT Reading

Get a 36 on ACT Science

How to Improve Your Low ACT Score

Get a 24 on ACT English

Get a 24 on ACT Math

Get a 24 on ACT Reading

Get a 24 on ACT Science

Stay Informed

Get the latest articles and test prep tips!

Follow us on Facebook (icon)

Ellen has extensive education mentorship experience and is deeply committed to helping students succeed in all areas of life. She received a BA from Harvard in Folklore and Mythology and is currently pursuing graduate studies at Columbia University.

Ask a Question Below

Have any questions about this article or other topics? Ask below and we'll reply!

Argument Essay: Evidence

9 min read • june 18, 2024

Stephanie Kirk

Stephanie Kirk

We aren’t sure where it started, but many teachers use REHUGO to help students find evidence on the Argument FRQ .  This acronym provides a quick check that can help you build logical evidence that supports your claim .

  • R - Reading - Something you have read, fiction or nonfiction, that connects the given topic.
  • E - Entertainment - A movie or song with dialogue or lyrics that present related ideas.
  • H - History - An event, document, speech, or person from history that aligns with the given topic.
  • U - Universal Truths - A common maxim or socially-accepted quote people tend to accept as truth.
  • G - Government - A national or international current event or governmental situation related to the topic.
  • O - Observations - Any cultural, technical, or societal trend that relates to the topic.

Suggested Guided Questions for the Argument FRQ

Now that you have a better understanding of the Argument FRQ’s expectations and scoring, let’s visit a sample prompt and add a few guided questions that you can use to help plan your own writings.

In his book   Canadian journalist Malcolm Gladwell (born 1963) writes: “To assume the best about another is the trait that has created modern society. Those occasions when our trusting nature gets violated are tragic. But the alternative—to abandon trust as a defense against predation and deception—is worse.”Write an essay that argues your position on the importance of .

Guided Question 1: What does the prompt say? 📝

  • Why do I do this?  Understanding the concept or idea presented by the prompt is vital to planning a response that thoroughly addresses the prompt and stays on topic throughout.  This is where you are going to  BAT the PROMPT .
  • Background : Gladwell asserts that society should trust each other in order to continue to be productive. Assuming the best about each other presents a better outcome than assuming the worst about each other.
  • Advice : The new stable prompt wording does not give much advice, but you should revisit advice you learned in class or from us as Fiveable -- things like using Toulmin to plan your response and planning modes of development that help progress your reasoning.
  • Task : Write an essay giving your position about the importance of trust. Specifically, is Gladwell right or wrong? And why? 🎥 Watch: AP Lang -   Argumentation, Part I: It's a Trap!

Rhetorical Situation : When writing for AP Lang, it is important to consider the rhetorical situation and write in a manner that demonstrates an understanding of all elements of that situation. 

  • Context - the historical, social, and cultural movements in the time of the text
  • Exigence - the urgency that leads to an action
  • Purpose - the goal the speaker wants to achieve and the desired audience movement
  • Persona - the “mask” shown to his/her audience

how to write essay for ap lang

Image Courtesy of Dana Anderson,   Writing Unleashed

Guided Question 2: What do I think? 💡

Why do I do this?   Taking a moment to brainstorm ideas can help organize thoughts and build an outline that you can revisit if you lose your train of thought in the stress of timed writing.

What does it look like?   This might just be stream-of-consciousness in your head, cloud diagrams, or even bulleted notes on the side of your prompt, but it needs to end with a clear position statement you can use for your thesis statement . For example: Trust is important. It does suck to get betrayed though but having a positive outlook creates positive results. Thinking the worst makes people act negatively because they project in a way that leads toward the worst response. ⬇️

  • Thesis Statement: Although some people believe humanity seems self-interested, a trusting nature enables individuals to focus on the positive and treat others with the respect that foster positive interrelationships.

Guided Question 3: What evidence can I use? 🤔

  • Why do I do this?   Revisit REHUGO and use Toulmin to plan your body paragraphs based on the thesis statement you came to when brainstorming ideas.
  • Modes of Development:  When writing, it is helpful to arrange the overall essay and its parts in a way that aligns with the purpose.  Consider these basic modes and how you might use them in writing an argumentative essay.- Cause and Effect attempt to follow the chain of events and establish causation. The description brings imagery and details into a text so that it set up the tone and ensures the reader can follow the mood.- Classification allows the speaker to categorize things in a way that guides the reader to follow the line of reasoning.- Comparison , looking at the similarities and differences, helps to analyze the intricate details of a given topic.  Because this describes differing elements, it may be structured by the element or by the characteristic.- Definition  works to explain what something is or is not. By defining the subject being discussed, the speaker is able to control the thinking about that subject.  Because this helps to clarify the topic, it is generally used in the introductory section of argumentation.- Exemplification  is used when explaining the topic or situation by giving examples to help lead the audience to the desired conclusion.- Narration  tells a story or gives an anecdote to help illustrate the point.- Process Analysis serves to explain the process by which something is done.
  • What does it look like?   I always use a version of the T-chart because one side is my evidence and the other side helps me consider multiple perspectives.  You may not have an idea in all areas, and you may have multiple ideas in one area. Try to time yourself so that you get plenty of strong evidence without wasting too much time. | Supports 🏗️ | Thoughts 💭 | | --- | --- | | ~ R -  Trust in Society by Cook points out that we only realize the importance of trust when there’s a breakdown.~ R - Essay “Importance of Trust” from class said that trust is not easy but it is what builds the foundation of a relationship and drives all actions between sides.~ E -  The 2011 movie  Trust shows that too much trust can get you hurt or killed.~ E - The media has so much bias it can’t be trusted… ~ H - Revolutionary War - trusted founding fathers and God’s position resulting in breaking free from Britain~ U - Trust - People should be trusted until proven otherwise~ G - A criminal escaped in our town and was on the loose; we trusted police to do their job rather than resort to vigilante justice~ G - Trust the president will have our best interests at heart -- most appear to have done that~ O - App-based dating - relies on trusting the person you meet online before meeting in person  | > relationships between couples> trust in contractual agreements? Moral obligation for trust?> Counter?> Relied on trust and won with not much else to go ob> Ex, Parents, Teachers, Siblings > Trust helped keep us safe; job was well done when we were out of the way> Obvious exceptions; System of impeachment  |

🎥 Watch: AP Lang -   Review: Argument Body Paragraphs

PRO TIP:  What if you have more evidence for the other side? Well, you haven’t started writing just yet, so it isn’t too late to go back and revise the thesis statement. Sometimes this happens in looking for evidence, and that’s perfectly okay.  In fact, many times students will say they wrote an essay using evidence that went completely against what they felt in their head or heart just because they couldn’t put a logical argument on paper without getting too emotional.   Know your audience, and build your argument.

Guided Question 4: How should I effectively organize my response? 📈

  • Why do I do this?   This step helps to outline the response so that your ideas are organized before you start writing.  The general advice is to follow the structure of  Classical Argumentation , but there is no rule saying that must be done to score well on the rubric. 💯
  • What does it look like?   One way of doing this would be to mark numbers by ideas tracked and written in the brainstorm, but some do take a moment to build an outline with topic sentences.
  • Modes of Development:  When writing, it is helpful to arrange the overall essay and its parts in a way that aligns with the purpose.  Consider these basic modes and how you might use them in writing an argumentative essay.- Cause and Effect attempt to follow the chain of events and establish causation. The description brings imagery and details into a text so that it set up the tone and ensures the reader can follow the mood.- Classification allows the speaker to categorize things in a way that guides the reader to follow the line of reasoning.- Comparison , looking at the similarities and differences, helps to analyze the intricate details of a given topic.  Because this describes differing elements, it may be structured by the element or by the characteristic.- Definition works to explain what something is or is not. By defining the subject being discussed, the speaker is able to control the thinking about that subject.  Because this helps to clarify the topic, it is generally used in the introductory section of argumentation.- Exemplification is used when explaining the topic or situation by giving examples to help lead the audience to the desired conclusion.- Narration tells a story or gives an anecdote to help illustrate the point.- Process Analysis serves to explain the process by which something is done.
  • Start by creating  Toulmin Position Statements that can be used for topic sentences and then move into a writing plan. Here’s a sample for this prompt:

Sample Outline:

  • Revised Thesis: Although some people believe trust must be earned, maintaining a trusting nature is important because it enables society to focus on positivity and create positive interrelationships that lead to positive outcomes.
  • Universal truth
  • Observation
  • Entertainment - refute
  • Conclude: When considering the value of trust in society, it is clear that the benefits of granting trust far outweigh the consequences of withholding it.

Key Terms to Review ( 35 )

Fiveable

Stay Connected

© 2024 Fiveable Inc. All rights reserved.

AP® and SAT® are trademarks registered by the College Board, which is not affiliated with, and does not endorse this website.

Get your 2024 AP scores now.

AP English Language and Composition Writing Study Skills

Writing is central to the AP English courses and exams. Both courses have two goals: to provide you with opportunities to become skilled, mature, critical readers, and to help you to develop into practiced, logical, clear, and honest writers. In AP English, writing is taught as “process”—that is, thinking, planning, drafting the text, then reviewing, discussing, redrafting, editing, polishing, and finishing it. It’s also important that AP students learn to write “on call” or “on demand.” Learning to write critical or expository essays on call takes time and practice.

Here are some key guidelines to remember in learning to write a critical essay:

  • Take time to organize your ideas.
  • Make pertinent use of the text given to you to analyze.
  • Quote judiciously from the text to support your observations.
  • Be logical in your exposition of ideas.

If you acquire these skills—organizing ideas, marshalling evidence, being logical in analysis, and using the text judiciously—you should have little trouble writing your essays on the AP Exam. Practice in other kinds of writing—narrative, argument, exposition, and personal writing—all have their place alongside practice in writing on demand.

As you study and practice writing, consider the following points.

Reading directly influences writing skills and habits.

Reading and writing are intertwined. When you read what published authors have written you are immersed not just in their ideas, but in the pulsing of their sentences and the aptness of their diction. The more you read, the more that the rhythm of the English language will be available to influence your writing. Reading is not a substitute for writing, but it does help lay the foundation that makes good writing possible.

Writing is fun.

When you have penned what you think is a great sentence or a clean, logical paragraph, read it over to yourself out loud. Enjoy it. Delight in the ideas, savor the diction, and let the phrases and clauses roll around in your mind. Claim it as part of yourself. You may discover you have a voice worthy of respect.

A tip from E. M. Forster

He is reputed to have said that he never knew clearly what it was he thought until he spoke it; and once he had said it, he never knew clearly what it was that he said until he had written it down. Then, Forster noted, he could play with it and give it final form. Be like Forster: think, speak, write, analyze your writing, then give it final shape.

Write purposefully with rhetorical awareness.

When you write, fashion your text with awareness of key rhetorical elements. What is the message of your text? How do you intend to convey your message to your particular audience? Give shape to your thinking with language that enlightens your readers and lets you achieve your aims.

Pay close attention to the task verbs used in the free-response questions. Each one directs you to complete a specific type of response. Here are the task verbs you’ll see on the exam:

  • Analyze: Examine methodically and in detail the structure of the topic of the question for purposes of interpretation and explanation.
  • Argue your position: Formulate a claim and support it with evidence.
  • Read: Look at or view printed directions and provided passages.
  • Synthesize: Combine different perspectives from sources to form a support of a coherent position.
  • Write: Produce a response in writing.

What are your chances of acceptance?

Calculate for all schools, your chance of acceptance.

Duke University

Your chancing factors

Extracurriculars.

how to write essay for ap lang

Ultimate Guide to the AP English Language and Composition Exam

Do you know how to improve your profile for college applications.

See how your profile ranks among thousands of other students using CollegeVine. Calculate your chances at your dream schools and learn what areas you need to improve right now — it only takes 3 minutes and it's 100% free.

The AP Language and Composition exam is one of the most popular exams taken year after year. In fact, in 2019, over 10% of the more than five million students who took AP exams took the Language and Composition test. AP Lang is most often taken by high school juniors, many of whom go on to take the AP English Literature exam their senior year. Plenty of seniors and even sophomores take this test too though, contributing to its popularity. If you’re planning to take the AP Language and Composition exam, whether you have taken the class or self-studied, look no further. Here’s our complete guide to the AP Lang exam, full of expert tips and free study resources.

When is the AP Language and Composition Exam?

On Wednesday, May 6, at 8 am, the College Board will hold the 2020 AP Language and Composition exam. For a comprehensive listing of all the AP exam times and AP score distributions from 2019, check out our post 2020 AP Exam Schedule: Everything You Need to Know .    

About the AP Language and Composition Exam

The AP Language and Composition exam is based primarily on the study of rhetoric, wherein an author attempts to persuade, inform, or motivate an audience using established techniques. The College Board encourages students who are interested in studying and writing various kinds of analytic or persuasive essays on nonliterary topics to take this course. It tests students on their reading comprehension, rhetorical analysis, synthesis of information, and written argumentation.

Big Ideas: The AP Language and Composition exam is built on a foundation of four big ideas. Big ideas are threads that run throughout the AP Language and Composition course that are vital for making connections and developing a deeper understanding of concepts found within it. The four big ideas are: 

  • Rhetorical Situation: Understanding what an author is communicating, how they convey that message, and what the impact of their rhetorical strategies are.
  • Claims and Evidence: Making claims and justifying them, while acknowledging or responding to opposing arguments. 
  • Reasoning and Organization: Guiding a reader’s understanding of text through its organization and the development of its argument. 
  • Style: The stylistic choices writers make and their impact. 

Course Skills: Along with exploring and connecting concepts with big ideas, students will develop eight course skills—four sets of two paired reading and writing skills—necessary for analyzing and composing arguments. The course skills and the weight they’re given on the multiple-choice section of the AP Language and Composition exam are: 

Rhetorical Situation – Reading Explain how writers’ choices reflect the components of the rhetorical situation. 11%-14%
Rhetorical Situation – Writing Make strategic choices in a text to address a rhetorical situation. 11%-14%
Claims and Evidence – Reading Identify and describe the claims and evidence of an argument. 13%-16%
Claims and Evidence – Writing Analyze and select evidence to develop and refine a claim. 11%-14%
Reasoning and Organization – Reading Describe the reasoning, organization, and development of an argument. 13%-16%
Reasoning and Organization – Writing Use organization and commentary to illuminate the line of reasoning in an argument. 11%-14%
Style – Reading Explain how writers’ stylistic choices contribute to the  purpose

of an argument.

11%-14%
Style – Writing Select words and use elements of composition to advance an argument. 11%-14%

  About the AP Language and Composition Exam Content

The Language and Composition exam is one of the longer AP exams, clocking in at 3 hours and 15 minutes from start to finish. The Language and Composition exam is structured in two sections—one featuring multiple-choice, the other free-response questions. 

Section 1: Multiple Choice 

1 hour | 45 questions | 45% of score

There have been some changes to the AP Language and Composition Exam for 2020. The first section remains one hour long and is still worth 45% of your score, but the number of questions has shrunk from 52-55 to 45. The variance in types of questions asked is also now more clearly defined—questions are now presented in 5 sets with 23-25 reading questions (reading and analyzing nonfiction texts) and 20-22 writing questions (“read like a writer” and consider revisions to stimulus texts), both of which will use shorter stimuli than previous exams. Below is the structure of the multiple-choice section of the AP Language and Composition exam. 

1 11-14 Reading Skills
2 11-14 Reading Skills
3 7-9 Writing Skills
4 7-9 Writing Skills
5 4-6 Writing Skills

Sample of a multiple-choice reading question: 

ap lang sample questions

Sample multiple-choice writing question: 

ap lang sample question

Section 2: Free Response

2 hour 15 minutes | 3 questions | 55% of score

The second section takes 2 hours and 15 minutes to complete and consists of 3 free response questions worth 55% of your score. These prompts are each of a different type: one synthesis question, one passage analysis, and one argumentative essay.

Synthesis Question: The synthesis question asks students to consider a scenario and then formulate a response to a specific element of it using at least three accompanying sources for support. Sources used in the essay need to be cited to be considered legitimate.  

Sample synthesis free response question: 

ap lang sample question

Analysis Question: The analysis question asks students to read a short passage and analyze and discuss various devices used by the author, such as strategies, argumentative techniques, or motivations. 

Sample analysis free response question: 

ap lang sample question

Argument Question: The argument question gives a position in the form of an assertion from a documented source and asks students to form their own argument to defend, challenge, or qualify it using supporting evidence. 

Sample argument free response question: 

ap lang sample question

The format of the free response section is unchanged this year; however, the scoring has shifted from a holistic rubric to an analytic rubric. The new rubric hasn’t been released, but you can gain insight into what type of answers the College Board is looking for by reading the sample free response questions found in the AP Language and Composition Course and Exam Description . 

AP Language and Composition Score Distribution, Average Score, and Passing Rate

AP Language and Composition    9.9% 18.2% 26.2% 31.2% 14.5%

In 2019, 54.3% of the students who took the AP English Language and Composition exam received a score of 3 or higher. Only 9.9% of students who took the exam achieved the top score of 5, and 14.5% of students who took the exam scored a 1. That said, students take the course seriously and prepare diligently will often find that the test is not as difficult as the results indicate.

 If you’re curious about other score distributions, see our post Easiest and Hardest AP Exams .

how to write essay for ap lang

Best Ways to Study for the AP Language and Composition Exam

Step 1: assess your skills.

Take a practice test to assess your initial knowledge. Though the College Board AP Language and Composition website provides a number of sample test questions, it does not provide a complete sample test. You can find a practice test in many of the official study guides, and some even include a diagnostic test to act as your initial assessment. Varsity Tutors offers a handful of free AP Language and Composition diagnostic tests on its website. You’ll also find a free practice exam from College Countdown to use for your assessment. 

Once you have taken some kind of formative assessment, score it to identify your areas of strength and areas in need of improvement. It can be helpful to have a teacher or friend score your free-response essays, as these are a bit more subjective than the multiple-choice section. With an accurate formative assessment, you’ll have a better idea of where to focus your studying efforts.

Step 2: Know Your Material

In the case of AP English Language and Composition, this means focusing on your reading and writing skills.

When reading, make sure to preview important elements such as the title, author’s name, and any other information available like the table of contents or introduction. As you read, make sure to stop periodically to consider the main ideas and the way the author supports them. Underline important evidence as you go. Reread complex or important sentences.      

One consultant to the College Board writes about the “SOAPSTone” approach to reading, which is an acronym for a series of questions that students should ask themselves when analyzing a 

piece of prose. The questions are:

  • Who is the Speaker?
  • What is the Occasion?
  • Who is the Audience?
  • What is the Purpose?
  • What is the Subject?
  • What is the Tone?

For more about using this technique, read about it on the College Board website .

Writing high-quality free-response essays takes practice and time. Make sure to organize your ideas using a rough outline before you begin writing. Use direct evidence from the text to support your ideas, and quote judiciously with correct citations. As you’re writing, be aware of rhetorical elements and use them effectively.

For more specific information about the test, consider using a formal study guide, such as Barron’s AP English Language and Composition, 7th Edition or the Princeton Review’s Cracking the AP English Language & Composition Exam 2020, Premium Edition . 

Alternatively, there are many online study resources available. Some AP teachers have even published their own study guides or review sheets online, like this AP Lang guide by Mrs. Smith at Pinnacle High School.

Another way to study is to use one of the recently-developed apps for AP exams. These are a great way to get practice questions in while on-the-go. Make sure you read reviews before choosing one, as their quality varies widely. This AP Lang app by Varsity Tutors has decent reviews, and might be worth checking out.

Step 3: Practice Multiple-Choice Questions

Once you have your theory down, test it out by practicing multiple-choice questions. You can find these in most study guides or through online searches. Here are some AP Lang practice questions and tests , and more are available in the College Board’s course description .

Try to keep track of which areas are still tripping you up, and go back over this theory again. Keep in mind, the key to answering questions correctly is understanding the passage, so practice active reading skills as you tackle the multiple-choice questions. This includes underlining, mouthing words, and circling key points. Remember, the answer will always be found in the text.

Step 4: Practice Free-Response Essays

As indicated on your exam, it is recommended that you spend 15 minutes reading the question, analyzing, and evaluating the sources, and 40 minutes writing your response. Try to stick to this timeline when practicing your free-response essays to see if it works for you. You do not have to follow it on exam day, but having a good idea of how much time it typically takes for you to plan and write will be an advantage.

As you tackle your open responses, identify what each is asking you to do. When asked to synthesize, you know you will be taking pieces of evidence from multiple sources to form a single argument. Use specific examples and make them stand out by explicitly stating, “For example…” or “As Source C indicates in paragraph 3…” Also, be sure to cite your sources appropriately while writing.

When writing an analysis of rhetorical strategies used, first consider the elements of SOAPSTone as discussed above. Also consider the five canons of rhetoric . This means thinking about the author’s invention, arrangement, and style. Memory and delivery are obviously less apparent in written pieces, but their roles in a speech are still important. As you read, try to underline specific places that highlight relevant examples.   

Finally, when writing your own persuasive argument, support your ideas with concrete examples from current events, literature, etc. Try to vary your sources to build credibility and address counterpoints to craft an even stronger response.

As you prepare for the writing portion of your exam, be sure to review how your free responses will be scored. The College Board supplies free response questions and authentic scored student responses with written explanations dating back to 1999; these are an invaluable tool for this exercise.

Step 5: Take Another Practice Test

Take another practice test to evaluate the progression of your knowledge, as well as identify persistent areas of weakness. Study.com offers a free online practice AP Language and Composition exam . Over time, you should begin to notice areas in which your studying should be increased and those which you are strong in. Repeat the above steps if time permits to incrementally increase your score. 

Step 6: Exam Day Specifics

If you’re taking the AP course associated with this exam, your teacher will walk you through how to register. If you’re self-studying, check out our blog post How to Self-Register for AP Exams .

For information about what to bring to the exam, see our post What Should I Bring to My AP Exam (And What Should I Definitely Leave at Home)?

Want access to expert college guidance — for free? When you create your free CollegeVine account, you will find out your real admissions chances, build a best-fit school list, learn how to improve your profile, and get your questions answered by experts and peers—all for free. Sign up for your CollegeVine account today to get a boost on your college journey.

For more guidance about the AP exams, check out these other informative articles: 

2020 AP Exam Schedule

How Long is Each AP Exam?

Easiest and Hardest AP Exams

Related CollegeVine Blog Posts

how to write essay for ap lang

  • [email protected]
  • (650) 338-8226

Cupertino, CA

AdmissionSight Logo

  • Our Philosophy
  • Our Results
  • News, Media, and Press
  • Common Application
  • College Application Essay Editing
  • Extracurricular Planning
  • Academic Guidance
  • Summer Programs
  • Interview Preparation

Middle School

  • Pre-High School Consultation
  • Boarding School Admissions

College Admissions

  • Academic and Extracurricular Profile Evaluation
  • Senior Editor College Application Program
  • Summer Program Applications
  • Private Consulting Program
  • Transfer Admissions
  • UC Transfer Admissions
  • Ivy League Transfer Admissions

Graduate Admissions

  • Graduate School Admissions
  • MBA Admissions

Private Tutoring

  • SAT/ACT Tutoring
  • AP Exam Tutoring
  • Olympiad Training

Research Programs

  • Science Research Program
  • Humanities Competitions
  • Passion Project Program
  • Ad Hoc Consulting
  • Athletic Recruitment
  • National Universities Rankings
  • Liberal Arts Colleges Rankings
  • Public Schools Rankings

Acceptance Rates

  • University Acceptance Rates
  • Transfer Acceptance Rates
  • Supplemental Essays
  • College Admissions Data
  • Chances Calculator
  • GPA Calculator

National Universities

  • College Acceptance Rates
  • College Overall Acceptance Rates
  • College Regular Acceptance Rates
  • College Early Acceptance Rates
  • Ivy League Acceptance Rates
  • Ivy League Overall Acceptance Rates
  • Ivy League Regular Acceptance Rates
  • Ivy League Early Acceptance Rates

Public Schools

  • Public Schools Acceptance Rates
  • Public Schools Overall Acceptance Rates
  • Public Schools Regular Acceptance Rates
  • Public Schools Early Acceptance Rates

Liberal Arts

  • Liberal Arts Colleges Acceptance Rates
  • Liberal Arts Colleges Overall Acceptance Rates
  • Liberal Arts Colleges Regular Acceptance Rates
  • Liberal Arts Colleges Early Acceptance Rates

AdmissionSight Logo

The Ultimate Guide to Acing the AP English Language and Composition Exam

how to write essay for ap lang

By Eric Eng

how to write essay for ap lang

Are you a high school student taking the AP English Language and Composition Exam? Are you seeking tips and strategies to help you ace this challenging test? Look no further! In this comprehensive guide, we will walk you through everything you need to know to succeed on the AP English Language and Composition Exam.

What is the AP English Language and Composition exam?

how to write essay for ap lang

The AP English Language and Composition Exam is a standardized test administered by the College Board . As the name suggests, it assesses your skills in English language and composition. The exam measures your ability to read, analyze, and write critically about various texts.

The AP English Language and Composition Exam is typically taken by high school students who have completed a course in advanced English. Colleges and universities widely recognize it as a way to demonstrate your proficiency in English language and composition.

Why is the AP English Language and Composition exam important?

This exam holds significant importance for high school students, educational institutions, and the broader academic landscape for several reasons:

how to write essay for ap lang

College Credit and Advanced Placement : Passing the AP English Language and Composition exam can earn students college credit, which allows them to bypass introductory English courses in college. This not only saves time but also reduces the cost of college education . It can also open up opportunities to enroll in more advanced coursework sooner.

Academic Rigor : The exam is designed to be a challenging assessment of a student’s language and composition skills. Successfully navigating this challenge reflects students’ ability to engage with complex texts, analyze rhetoric, and construct persuasive arguments.

College Admissions: High performance on the AP English Language and Composition exam can enhance a student’s college application. It signals to college admissions committees that the student is well-prepared for the demands of college-level work, which can be a deciding factor in the competitive college admissions process.

Transferable Skills: The skills acquired in AP English Language and Composition, including critical reading, rhetorical analysis, and effective writing, are highly transferable. These skills are valuable not only in other academic pursuits but also in future careers where communication and research are essential.

Well-Rounded Education : AP courses, including English Language and Composition, are part of a broader effort to provide a well-rounded, advanced education. They encourage students to think critically, engage with complex texts, and communicate effectively, fundamental skills applicable across various disciplines.

Higher Educational Standards: The exam encourages high schools to maintain rigorous academic standards. By offering AP courses and encouraging students to take the associated exams, high schools emphasize the importance of challenging coursework.

Teacher Professional Development: The preparation and teaching of AP courses often lead to professional development for educators. This, in turn, benefits students by providing access to experienced and well-trained teachers.

Curriculum Development: The AP English Language and Composition course and exam help shape the curriculum for high school English courses. They set standards for what students should know and be able to do, which can improve overall educational quality.

National Consistency: The AP program is a nationally recognized standard for advanced coursework. It provides a consistent benchmark for student achievement, making it easier for colleges and universities to assess the qualifications of incoming students.

Life-Long Learning: The skills cultivated in AP English Language and Composition extend beyond academics. They equip students to be more effective and persuasive communicators in their personal and professional lives, promoting lifelong learning and engagement with various texts and ideas.

How long is the AP English Language and Composition Exam?

The AP English Language and Composition exam is 3 hours and 15 minutes long and consists of two sections: a multiple-choice section and a free-response section. The multiple-choice section has 45 questions and lasts for 1 hour, while the free-response section has 3 questions and lasts for 2 hours and 15 minutes (including a 15-minute reading period). The multiple-choice section accounts for 45% of the exam score and includes 5 sets of questions that assess reading and writing skills.

Topics Covered in the AP English Language and Composition Exam

The AP English Language and Composition Exam covers a wide range of topics related to language and composition. It is designed to test your understanding and mastery of various aspects of English language and composition. By exploring these topics, you will develop valuable skills essential for effective communication and critical thinking.

One of the critical areas you should familiarize yourself with is rhetorical analysis. Rhetorical analysis involves examining and evaluating the strategies and techniques writers use to convey their ideas and persuade their audience. This includes analyzing rhetorical devices such as ethos, pathos, and logos, as well as the overall structure and organization of a text.

Another important area covered in the exam is argumentative writing. Argumentative writing requires presenting a clear and well-supported argument on a given topic. This involves not only stating your position but also providing evidence, reasoning, and counterarguments to strengthen your argument and address potential objections.

A woman with her things

In addition to rhetorical analysis and argumentative writing, the exam also includes a section on synthesis. Synthesis combines multiple sources to create a cohesive and well-supported argument or analysis. This requires you to critically evaluate and integrate information from various texts, demonstrating your ability to synthesize ideas and present a coherent perspective.

Furthermore, the exam tests your reading comprehension skills. You will be presented with various passages and asked to analyze and interpret their meaning. This includes understanding the main ideas, identifying supporting evidence, and evaluating the author’s tone and purpose.

Lastly, the exam assesses your ability to identify rhetorical devices. Rhetorical devices are techniques writers use to enhance their writing and engage their audience. These devices can include similes, metaphors, hyperbole, and many others. By recognizing and analyzing these devices, you will gain a deeper understanding of how writers use language to convey meaning and create impact.

Overall, the AP English Language and Composition Exam covers various crucial topics for effective communication and critical thinking. By studying and practicing these areas, you will not only be well-prepared for the exam but also develop valuable skills that will benefit you in college and beyond.

What is on the AP English Language and Composition exam?

The AP English Language and Composition exam is designed to test students’ understanding of the literary concepts covered in the course units, as well as their ability to analyze texts and develop written arguments based on their interpretations. The exam has two sections: a multiple-choice section and a free-response section.

The exam assesses students’ understanding of the literary concepts covered in the course units, as well as their ability to analyze texts and develop written ideas based on their interpretations. The exam questions assess the course concepts and skills outlined in the course framework. The College Board provides a detailed course and exam description (CED) that outlines what students are expected to know and be able to do on the exam .

1. Rhetorical Situation: Reading Explain how writers’ choices reflect the components of the rhetorical situation. 11%–14%
2. Rhetorical Situation: Writing Make strategic choices in a text to address a rhetorical situation. 11%–14%
3. Claims and Evidence: Reading Identify and describe the claims and evidence of an argument. 13%–16%
4. Claims and Evidence: Writing Analyze and select evidence to develop and refine a claim. 11%–14%
5. Reasoning and Organization: Reading Describe the reasoning, organization, and development of an argument. 13%–16%
6. Reasoning and Organization: Writing Use organization and commentary to illuminate the line of reasoning in an argument. 11%–14%
7. Style: Reading Explain how writers’ stylistic choices contribute to the purpose of an argument. 11–14%
8. Style: Writing Select words and use elements of composition to advance an argument. 11–14%

Multiple-choice section

Here, you will encounter various passages from different genres, including fiction, nonfiction, and poetry. These passages are carefully selected to test your ability to comprehend and interpret the author’s intentions, as well as your skills in analyzing their rhetorical strategies. You will be required to answer questions that delve into the nuances of the texts, exploring themes, tone, figurative language, and other literary devices.

Tackling the multiple-choice section

Here are some tips on how to best tackle the multiple-choice section of the AP English Language and Composition exam:

  • Read the questions before reading the passage: This will help you focus on the information that is relevant to the questions and save time.
  • Annotate the passage: Underline or highlight important information, such as the author’s main argument, tone, and rhetorical devices used.
  • Identify the author’s purpose: Determine why the author wrote the passage and what message they are trying to convey.
  • Pay attention to context: Consider the passage’s context, including the period, audience, and historical events that may have influenced the author’s writing.
  • Eliminate wrong answers: Use the process of elimination to eliminate incorrect solutions, which will increase your chances of selecting the correct answer.
  • Practice with sample questions: Use practice questions from the College Board or other sources to familiarize yourself with the types of questions that will be on the exam.

Remember that the multiple-choice section of the exam assesses your reading and writing skills, so it’s essential to focus on these skills when preparing for the exam. Use the charts provided by the College Board to identify the skills that will be assessed and concentrate your studying on those skills.

  • Pace yourself: The multiple-choice section has 45 questions and lasts for 1 hour, so you should aim to spend no more than 1-2 minutes on each question.
  • Skip difficult questions: If you encounter a difficult question, don’t spend too much time on it. Instead, skip it and come back to it later if you have time.
  • Answer easy questions first: Start with the ones you find easy and move on to the more difficult ones.
  • Use the process of elimination: Eliminate incorrect answers to narrow down your choices and increase your chances of selecting the correct answer.
  • Don’t spend too much time on annotations: While it’s important to annotate the passage, don’t spend too much time on it. Focus on the information that is relevant to the questions.
  • Keep track of time : Use a watch or timer to keep track of time and make sure that you are on track to finish the section on time.

Remember that the multiple-choice section assesses your reading and writing skills, so it’s essential to focus on these skills when preparing for the exam. Use practice questions from the College Board or other sources to familiarize yourself with the types of questions that will be on the exam and practice managing your time effectively.

how to write essay for ap lang

Free-response section

The free-response section of the exam is divided into three tasks, each demanding a unique set of skills. The first task is the synthesis essay , where you will be presented with multiple sources on a specific topic. Your task is to synthesize these sources and develop a well-organized argument incorporating evidence from the texts. This task not only tests your ability to comprehend and evaluate multiple perspectives but also challenges you to construct a cohesive argument that showcases your analytical and writing skills.

The second task in the free-response section is the rhetorical analysis essay. In this task, you will be given a passage and asked to analyze the author’s rhetorical strategies. This requires a deep understanding of rhetorical devices such as ethos, pathos, and logos, as well as the ability to identify and explain the impact of these strategies on the intended audience. Your analysis should demonstrate your ability to deconstruct the author’s persuasive techniques and effectively communicate your insights.

The final task in the free-response section is the argumentative essay. Here, you will be provided with a prompt that presents a specific issue or topic. Your task is to construct a persuasive argument, supported by evidence and reasoning, that effectively addresses the prompt. This task assesses your ability to articulate a clear and logical argument while demonstrating your command of rhetorical devices and persuasive techniques.

Overall, the AP English Language and Composition Exam is a rigorous assessment that challenges your reading, analysis, and writing skills. By mastering the ability to comprehend complex texts, analyze rhetorical strategies , and construct persuasive arguments, you will be well-prepared to excel on this exam and in future academic pursuits.

Here are some sample questions and answers from 2023’s exam so you’d know what to expect:

samples from AP English Language and Comp

Tackling the free-response section

The free-response section of the AP English Language and Composition exam requires students to write three essays that address three distinct tasks: synthesis, rhetorical analysis, and argument. Here are some tips on how to best tackle this section:

  • Read the prompts carefully: Ensure you understand what each prompt asks you to do before writing.
  • Plan your essays: Spend a few minutes planning and outlining your pieces before you start to write them. This will help you organize your thoughts and address all aspects of the prompt.
  • Use evidence: Use evidence from the texts provided to support your arguments. Be sure to cite your sources and explain how they support your thesis.
  • Address counterarguments: Identify counterarguments to your position and address them within your essay. This not only helps strengthen your place but also shows that you can consider multiple perspectives.
  • Be concise and clear: Write clearly and concisely, using precise language and avoiding unnecessary words or phrases.
  • Manage your time: The free-response section has 3 questions and lasts 2 hours and 15 minutes (including a 15-minute reading period), so make sure you manage your time effectively. Aim to spend no more than 40-45 minutes on each essay.

Remember that the free-response section assesses your ability to analyze texts and develop written arguments based on your interpretations. Use the charts provided by the College Board to identify the skills that will be assessed and focus your studying on those skills. Use practice questions from the College Board or other sources to familiarize yourself with the types of questions that will be on the exam and practice managing your time effectively.

AP English Language and Composition Scoring Breakdown

The AP English Language and Composition exam is scored on a scale of 1 to 5, with each score representing a different level of performance. Here is a breakdown of what each score means:

  • A score of 5 : This is the highest score a student can achieve on the AP English Language and Composition exam. A score of 5 indicates the student has demonstrated exceptional mastery of the subject matter. They have not only effectively analyzed and synthesized information from multiple sources but have also crafted persuasive, well-structured essays. This score often earns college credit and advanced placement in introductory English courses.
  • A score of 4 : A score of 4 is considered very good. It signifies a high level of competence in the subject. Students who earn a 4 have demonstrated a strong understanding of rhetoric, argumentation, and language usage. Their essays are well-crafted and persuasive, and they typically earn college credit and advanced placement.
  • A score of 3 : A score of 3 is considered passing and reflects a satisfactory level of performance. Students who earn a 3 have demonstrated a competent understanding of the subject matter and have produced essays that meet the basic criteria for the exam. Many colleges and universities will grant college credit or advanced placement for a score of 3, but this can vary by institution.
  • A score of 2 : A score of 2 is considered inadequate. It indicates that the student’s performance on the exam was below the standard expected for passing. Essays may be poorly constructed, lack practical analysis, or fail to address the prompts adequately. Earning college credit or advanced placement is unlikely, with a score of 2.
  • A score of 1 : A score of 1 is the lowest possible score and signifies a severe deficiency in performance. Essays at this level are typically significantly flawed, with serious flaws in analysis, argumentation, or language usage. A score of 1 is generally not considered passing, and college credit or advanced placement is unlikely.

How hard is the AP English Language and Composition exam?

how to write essay for ap lang

The difficulty of the AP English Language and Composition Exam can vary from student to student. Some students find the exam challenging, while others find it relatively manageable. However, with proper preparation and practice, anyone can improve their performance on the exam.

One of the reasons why the AP English Language and Composition Exam can be challenging is because it requires critical, solid reading and analytical skills. You need to quickly identify and analyze key elements of a text, such as the author’s purpose, tone, and rhetorical strategies.

In addition, the free-response tasks require you to write three essays within a limited time. This requires good time management and constructing well-organized and coherent arguments.

Furthermore, the exam also assesses your understanding of various literary devices and techniques. You need to be familiar with concepts such as simile, metaphor, personification, and irony, as well as their effects on the overall meaning and impact of a text.

Moreover, the exam often includes passages from different periods and genres, ranging from classic literature to contemporary non-fiction. This means that you need to have a broad knowledge of various literary works and be able to make connections between them.

Additionally, the exam may require you to analyze visual and multimedia texts, such as advertisements or political cartoons. This adds another layer of complexity to the exam, as you need to be able to interpret and evaluate the visual elements and their relationship to the written content.

Furthermore, the multiple-choice section of the exam can be challenging due to the nuanced and sometimes ambiguous nature of the questions. You must carefully analyze each answer choice and select the most appropriate option based on your understanding of the text.

Lastly, the exam also tests your ability to revise and edit written passages effectively. You need to be able to identify errors in grammar, punctuation, and sentence structure, as well as suggest improvements to enhance clarity and coherence.

The AP English Language and Composition Exam can be challenging due to its emphasis on critical reading, analytical skills, time management, knowledge of literary devices, and the ability to interpret various texts. However, with dedication, practice, and a solid understanding of the exam’s expectations, you can improve your performance and achieve success.

How to study for the AP English Language and Composition Exam

Preparing for the AP English Language and Composition Exam requires studying content, practicing skills, and familiarizing yourself with the exam format.

First and foremost, it is crucial to review the content and skills covered in the exam. This includes understanding rhetorical devices, argumentative techniques, and various types of writing. Use review books, online resources, and past exams to help you acquire the necessary knowledge.

When reviewing the content, it is essential to delve deep into the nuances of each topic. For example, understanding rhetorical devices involves not only recognizing them but also being able to analyze their impact on the reader. Take the time to explore different models of rhetorical devices in literature and real-world texts to strengthen your understanding.

Moreover, familiarize yourself with the different types of writing that may be assessed on the exam. This includes understanding the characteristics of persuasive essays, rhetorical analyses, and synthesis essays. By studying the specific requirements and expectations for each type of writing, you can better prepare to meet the exam’s demands.

Additionally, practice is vital to success on the AP English Language and Composition Exam. Take practice tests to get a feel for the format and types of questions you will encounter. This will help you become familiar with the timing constraints and develop effective strategies for approaching different questions.

When practicing, challenge yourself to go beyond simply answering the questions. Take the opportunity to analyze the passages provided, identify the author’s purpose, and evaluate the effectiveness of their argument. This will not only strengthen your critical thinking skills but also enhance your ability to craft well-reasoned responses.

Furthermore, consider enrolling in an AP English Language and Composition course. These courses are designed to prepare students specifically for the exam. They provide guided instruction, practice exams, and feedback from experienced teachers.

During the course, take advantage of the opportunity to engage in class discussions and peer review sessions. These activities can help you refine your writing skills, gain new perspectives, and learn from the experiences of your classmates.

Additionally, seek feedback from your teacher on your practice essays and assignments. Their insights can provide valuable guidance on areas where you can improve and help you develop a more robust understanding of the exam’s expectations.

Finally, remember that preparation for the AP English Language and Composition Exam is not just about acquiring knowledge and skills. It is also about building confidence in your abilities. Take the time to reflect on your progress and celebrate your achievements along the way. With dedication and perseverance, you can excel on the exam and showcase your mastery of the English language.

Test-Taking Tips for the Day of the Exam

The day of the AP English Language and Composition Exam can be nerve-wracking, but with these tips, you can approach the test with confidence:

First, it is crucial to get a good night’s sleep the night before the exam. Adequate rest is essential for optimal brain function and concentration. Establish a relaxing bedtime routine to ensure a peaceful night’s sleep. Avoid caffeine and electronic devices in the evening, as they can interfere with your ability to fall asleep.

In addition to a good night’s sleep, it is crucial to fuel your brain with a nutritious breakfast on the day of the exam. Choose food rich in protein, whole grains, and healthy fats to provide sustained energy throughout the test. Avoid sugary cereals or pastries that may cause a spike in blood sugar levels, leading to a subsequent crash.

Arriving at the testing center early is crucial to ensure a smooth start to the exam. Give yourself ample time to navigate any potential traffic or public transportation delays. By arriving early, you can familiarize yourself with the testing environment, which can help alleviate any test-day jitters.

Before leaving for the exam, double-check that you have all the necessary materials. This includes identification documents, such as a driver’s license or school ID, and writing utensils. It is always a good idea to bring extras, just in case one fails or runs out of ink during the exam.

Once you are seated and ready to begin, take a moment to read the instructions provided carefully. Pay close attention to specific guidelines or time limits for each exam section. Understanding the instructions thoroughly will help you approach the questions with confidence and clarity.

During the exam, it is vital to stay focused and manage your time effectively. Avoid getting stuck on difficult questions, and remember to pace yourself. If you encounter a challenging multiple-choice question, use the process of elimination to narrow down the options and increase your chances of selecting the correct answer.

When it comes to the essay portion of the exam, planning is vital. Take a few minutes to outline your thoughts and organize your ideas before you start writing. This will help you maintain a clear and logical structure throughout your essay, making it easier for the reader to follow your arguments.

Lastly, before submitting your essays, take the time to proofread them for errors and clarity. Check for spelling and grammar mistakes, as well as any areas that may require further clarification or elaboration. A well-polished essay will not only showcase your writing skills but also enhance the overall impact of your arguments.

When is the AP English Language and Composition Exam in 2024?

Unlike the SAT Subject Tests that are offered on multiple different days, all of the AP exams are only administered on one specific day of the calendar year. The exact date tends to fluctuate each year, although it generally falls within the first two weeks of May. The following AP English Language and Composition Exam is scheduled for Tuesday, May 14th, 2024, at 8 in the morning. For a complete list of the 2024 AP Exam dates, here’s a complete list:

United States Government and Politics Art History

Chemistry

Human Geography

Microeconomics

Seminar

Statistics

English Literature and Composition Comparative Government and Politics

Computer Science A

Chinese Language and Culture

Environmental Science

Psychology

European History

United States History

Macroeconomics

Spanish Literature and Culture

Calculus AB

Calculus BC

Italian Language and Culture

Precalculus

English Language and Composition African American Studies

Physics C: Mechanics

Physics C: Electricity and Magnetism

French Language and Culture

World History: Modern

Computer Science Principles

Music Theory

Spanish Language and Culture Biology

German Language and Culture

Physics 1: Algebra-Based

Latin

Physics 2: Algebra-Based

When do AP scores come out?

AP scores are typically released in early to mid-July of the year the exams were taken. The exact release date may vary slightly from year to year, but students who took their AP exams in May can generally expect their scores to become available.

If you are waiting for your AP scores, you can check for updates on the College Board’s official AP scores website. They will also send you an email notification when your scores are added to your score report. If you haven’t received your scores by mid-August, it’s recommended to contact AP Services for Students for assistance.

Post-Exam Tips

Once the AP English Language and Composition Exam is over, you can breathe a sigh of relief! However, there are still a few things you can do to ensure the best possible outcome:

  • Reflect on your performance and identify areas for improvement
  • Review your essays and analyze the feedback provided by the examiners, if available
  • Take note of any topics or types of questions that gave you trouble
  • Consider retaking the exam if you are not satisfied with your initial score
  • Celebrate your hard work and the knowledge you have gained

With this comprehensive guide, you can tackle the AP English Language and Composition Exam. Remember to study diligently, practice regularly, and stay calm and focused on the test day. Good luck!

how to write essay for ap lang

AdmissionSight is here for you.

If you’re preparing for college but are struggling with the admission process, the AdmissionSight team is here to help. We’ve helped students from across the country find their way into some of the best schools – including the most respected Ivy League Universities . After decades of practical experience, we’ve mastered the admission process. Whether you need help editing supplemental essays, choosing a summer program , or taking your AP exams, we have the expertise to help you succeed. We’re only happy when our students can achieve their academic goals, so you can count on us to do everything in our power to help you succeed. Feel free to contact us today to learn more about our services, our success stories, and how we can help you on your path toward academic success.

AdmissionSight

Want to assess your chances of admission? Take our FREE chances calculator today!

how to write essay for ap lang

Why College Admissions Isn’t Perfect

how to write essay for ap lang

US News Rankings

A person's hand writing in spiral notebook placed on a wooden desk.

The Personal Statement: The Holy Grail of College Admissions

Group of students taking ap classes in a classroom.

The Modern Day 4.0 and 1600 SAT Score Student Is No Longer Impressive

A woman writing a letter on a paper.

The Competitive Nature of College Admissions for Asian Americans

A professor talking to a student while they walk outside the classroom

The College Application

a woman sing laptop while "admission" word appears on screen

Our Comprehensive Approach

old school building

Ivy League Schools

a student daydreaming while sitting at the corner in library

How Early Should You Prepare for College?

how to write essay for ap lang

Featured in US News & World Report Best Colleges Publication

how to write essay for ap lang

Congratulations to AdmissionSight Students and their Acceptances!

A female student listening to the class lecture while holding a pen.

College Rejection

Group of students writing on their desks.

College Rankings

a fountain in front outside the building

College Consultants Could Make A Difference

A person holding a pen with a laptop in front.

College Admissions Scandal and Higher Education

group of friends, one male 2 females, sitting at a staircase

5 Factors to Consider Before Choosing Your Dream Study Destination

Young female student with laptop and headphones studying at table in cafe

Top Extracurricular Activities for Ivy Leagues

New York University

Where is NYU located?

how to write essay for ap lang

Where Is Williams Located?

Boston University signage

Where is Boston University Located?

how to write essay for ap lang

Where Is Barnard Located?

3 people talking to each other

Top 10 Undergraduate Business Programs

Claremont McKenna

Where is Claremont McKenna Located?

ivy league flags

Ivy League Rankings 2024

Students playing music in the school grounds.

10 Humanities Programs for High School Students

how to write essay for ap lang

Where is Georgia Tech located?

A woman shaking hands with her interviewer.

How to Qualify for National Merit Semifinalist 2025

how to write essay for ap lang

Where Is Harvey Mudd Located?

Academical Village at the University of Virginia (UVA)

Where is the University of Virginia located?

View of Wake forest campus

Where is Wake Forest located?

View of University of Michigan building

Where is the University of Michigan located?

Young man using a laptop in a table.

How to Update Caltech After Submitting Your Application

Leave a comment cancel reply.

Your email address will not be published. Required fields are marked *

Save my name, email, and website in this browser for the next time I comment.

Recent Articles

5 Factors to Consider Before Choosing Your Dream Study Destination

5 Factors to Consider Before...

Top Extracurricular Activities for Ivy Leagues

Top Extracurricular Activities for Ivy...

Where is NYU located?

10 Humanities Programs for High...

Where is Georgia Tech located?

How to Qualify for National...

Sign up now to receive insights on how to navigate the college admissions process..

admissionsight

Admissions Counseling

  • Academic & Extracurricular Profile Evaluation

Copyright © AdmissionSight 2024

Privacy Policy - Terms and Conditions

Get the Reddit app

No matter what course you are taking, we are a community that helps students earn college credit!

Guide for AP Lang Synthesis Essay

I have seen a few posts with ppl struggling to write the essay, so here's how I did it.

I'm not an English person, but this is how I would write question 1:

An intro (Skip this if you're running out of time).

Thesis statement including two of your sub-claims. I like a counterargument thesis, but a straightforward thesis gets the job done easier and you don't have to think too much about it.

Your topic sentence #1 which you should pull from your first sub-claim that you made in your thesis.

Now use a quote from a source which you should have already annotated and make sure you cite like this: (Source A). This would serve as one of your evidence.

Add commentary (at least 3 sentences minimum). Remember your commentary is your why that piece evidence proves your claim. Don't just summarize the source! Make sure you connect back to your thesis or sub-claim in your commentary.

Then add a transition that can help your flow on your essay it can be very simple. After that, add your second piece of quote from a source to serve as evidence. (similarly and in addition are great transitions).

Add commentary (at least 3 sentences).

Repeat steps 3-6 for your second body paragraph. Add a conclusion if you have time.

I highly recommend adding transitions after every evidence and commentary because this is really the only way to strengthen your line of reasoning. If you can't think of anything for your commentary I recommend adding a concession and refutation. This not only expands your argument, but makes your essay stand out more. Remember your readers are scoring this as a rough draft, so errors are ok.

What to do about a 3 on AP Lang?

I got a 3 on ap lang and a 5 on APWH. It was mostly because my English teacher was really nasty and didn’t show us how to properly put together an essay (most kids at my school taught by her fail this exam fwiw). I’m applying as a history major to ivies/top 20 unis. I got a >700 on the English section of the sat so I think that should even it out? Idk what to do because I don’t want colleges to think I failed the exam

You don’t have to report the score to most colleges including most Ivys/T20 and if you don’t they won’t care. So problem solved. (And even if you did a 3 on an AP isn’t going to be the sole reason you get in or don’t; Also, a 3 is not “failing” anyway.)

I’ll probably submit it, but basically they won’t hold it against me?

If they’re not going to give you credit, no need to submit.

And please don’t blame others for your score. People want to see that you take responsibility for your actions and results.

What to do about a 3 on ap lang

You move on. If your question is whether to list in the application for T20’s, probably not.

Now it’s time for some tough love:

Nobody wants to hear that, and it’s not an acceptable answer. Own your 3. You got a 3 because you didn’t prepare, not because the teacher was mean and nasty. There are adequate outside resources to do so. As you prepare for your AP exams next year, and for exams in college, you will need to take ownership for your own learning. Teachers are there to facilitate, not spoon-feed. And some teachers, professors, and bosses are better than others. You need to fill in the gaps as needed. Good luck

Ivy League schools and other “top 20” universities are reaches for pretty nearly all very strong students. As one example, a few years ago Stanford included an article on admissions in their alumni magazine. They estimated that 80% of their applicants are qualified to attend. They accept closer to 4% of applicants.

However, there are a large number of very good universities in the US. Make sure that you apply to safeties.

If you do get to attend an Ivy League university, or even if you don’t, you will find a lot of very good professors, and a few bad professors. You will need to deal with the bad professors.

Also, whether you get accepted to a “top 20” university or not, you will probably never know for sure why you either got accepted or didn’t. Those of us who did attend a highly ranked university can guess what got us accepted, but we are only guessing.

I’m worried it might look bad since I’m applying as a humanities major. It’s weird because I got a 5 on ap lit and ap physics last year which is harder than lang

You worry a lot - about everything.

You’re making a lot of assumptions about where you’re going, where you’re getting into, etc.

Your record is your record.

Just like the other thread - stop worrying - make sure you have two affordable and sure things - and after that, then take your shots.

You want to study History (a social science) - guess what, the “where” you study this likely won’t impact you in life.

You’re in high school - if you’re stressing this much now, you’ll be in for a long life.

Yes I know, it’s been tough lately because of my rapidly declining physical health and ocd. I just really want to get out of upstate ny

I’m sure you can get out of up state.

It’s a history degree - you’ll be fine no matter where you study. But take care of you first.

I’m not kidding when I say we never learned to put an essay together. We didn’t learn how to put together any of the essays from the test because half the time she would put on movies most of the time and get super angry when I would ask her about the exam content. The other half of the time she was not in class. English class is not my strong suit but I did prepare a lot. I don’t think my original post elaborated on this enough

You may consider a tutor then. Or summer class (it’s late). You will need to write multiple essays for the colleges you seek to attend.

Also you got a 3. So you did competent work. So I don’t think, personally, that you can’t write an essay. You wrote one and earned a decent score.

I understand all of this. BUT in an essay or in an interview you can not blame the teacher/classroom circumstances for the 3. Even if every fiber of your being wants to write about this or tell an interviewer about it, don’t.

I agree with others who say to not report the 3 to highly rejective schools. They will not assume you had a low score, plenty of students don’t take the tests so obviously have no score to report. Report the score to schools that give credit for the 3.

I also think that if you submit an ACT or SAT, even if you do submit a 3, it’s not gonna matter at all in the long run. It’s ONE number.

Like others are suggesting, I would just not submit this score if you don’t have to, and I wouldn’t worry about it generally.

That said, I think the substantive problem of not knowing basic essay structures and techniques seems pretty serious to me, particularly if you do end up at a highly selective college. Ideally you want to be a very confident essayist before you start at colleges like that, so I would be thinking carefully about how to achieve that result.

Here is a very important lesson in life that you need to keep in mind: Attending an Ivy League or “top 20” university does not matter. Keeping your stress to a manageable level and taking care of your health does matter.

I did attend two “top 20” universities (one for a bachelor’s degree, one for a master’s degree). I then spent my career working alongside people who attended more normal universities (some “top 100” and some not). I also spent most of my career working for a boss who had attended a more normal university. In the vast majority of cases, no one cares where you got your degree. They care that you get your job done. They care that you are a reasonable person to work with. They care whether you shower or bathe regularly. They do not even remember where you got your degree.

There are a lot of very good universities. There is generally a shortage of jobs for people with PhD’s in humanities. This means that a very wide range of universities can attract very strong professors.

The US puts way too much stress on high school students. Some of this comes from the pressure to try to attend a “big name” university. You should try not to fall into this trap. Relax. Do you best, but do not get stressed about minor issues such as a 3 on an AP test, or for that matter whether you get your bachelor’s degree from a “top 20” or “top 100” or “top 200” university.

Regarding not reporting an AP score, we did have a bit of experience with this. One daughter was just too stressed out senior year and skipped one of her AP tests. This of course meant that she did not report a score for that particular AP test. There were no consequences, other than not getting AP credit for that one course. She still was 6 for 7 in university admissions (or was it 7 for 8?), still went to a very good university and did well, and still got admitted to a very good graduate program (where she is studying now, with a bit less than one year to go). A year from now she will still be called “doctor”, and missing one AP test will be long forgotten (and a B- in organic chemistry was similarly not a problem). No one is perfect. We do just fine anyway.

:slight_smile:

Some general thoughts/advice to consider:

  • It’s not the end of the world.
  • You didn’t fail the test. You passed it.
  • Should you send the score to every college you apply to? Probably not. Just send it to the ones where the 3 would get you college credit.
  • Should you talk in a college app essay as to WHY you think you got a 3? NO.
  • Don’t apply to JUST top 20/ivies. Make sure you have MORE THAN ONE true safety school on your list. And by that, I mean a college that’s: (a) affordable; (b) you have high odds of getting accepted to; (c) has a campus vibe that you like; (d) has a major/program that you like.

If your teacher fails you, this is when you find other solutions rather than passively accept your fate. As others noted, there’s tutors, books, etc., but there’s also plenty of free options. If you had Googled “how to write a great a great ap language and composition essay” you would have found pages and pages of results with websites, blogs and videos (depending on your learning preference) offering concrete tips about how to structure and write the essays. Some paid, but many free. There’s also a pretty active discussion on Reddit. Obviously its too late this time, but its more of a life lesson. A lot of times you will need to be scrappy and proactive to achieve your goals. Life will more often throw up roadblocks instead of helpful mentors or people who tell you how to do something.

Anyway, again, this is not worth stressing about. A 3 on AP Lang is not going to affect your admissions anywhere. Your odds of getting into a T20 are low because they are low for everyone, regardless of your test scores. A single AP score will not make-or-break it. But there’s also a ton of places you are almost guaranteed to get into that are not in upstate NY.

how to write essay for ap lang

What's Project 2025? Unpacking the Pro-Trump Plan to Overhaul US Government

For several months, we received a flood of reader inquiries asking if project 2025 was a real effort to “reshape america.” here’s the answer., nur ibrahim, aleksandra wrona, published july 3, 2024.

  • Project 2025 is a conservative coalition's plan for a future Republican U.S. presidential administration. If voters elect the party's presumed nominee, Donald Trump, over Democrat Joe Biden in November 2024, the coalition hopes the new president will implement the plan immediately.
  • The sweeping effort centers on a roughly 1,000-page document  that gives the executive branch more power, reverses Biden-era policies and specifies numerous department-level changes.
  • People across the political spectrum fear such actions are precursors to authoritarianism and have voiced concerns over the proposal's recommendations to reverse protections for LGBTQ+ people, limit abortion access, stop federal efforts to mitigate climate change — and more.
  • The Heritage Foundation — a conservative think tank operated by many of Trump's current and former political allies — is leading the initiative. President Kevin Roberts once said  the project's main goals are "institutionalizing Trumpism" and getting rid of unelected bureaucrats who he believes wield too much political influence.
  • The Trump campaign's goals and proposals within Project 2025 overlap. However, the former president has attempted to distance himself from the initiative. In a July 5, 2024, post on Truth Social , he wrote: " I know nothing about Project 2025. I have no idea who is behind it. I disagree with some of the things they're saying and some of the things they're saying are absolutely ridiculous and abysmal. Anything they do, I wish them luck, but I have nothing to do with them."
  • In other words, it's unknown if, or to what extent, Trump's campaign is talking to leaders of the initiative. Many political analysts and the Biden administration believe Project 2025 is a good indication of Trump's vision for a second term.
Here at Snopes, the internet's premiere fact-checking site, we believe in unbiased, fact-driven reporting to help guide people's everyday lives. And when it comes to voting in elections, we hold that responsibility high. We call out candidates' mistruths, contextualize campaign claims and pull back the curtain on efforts shaping political parties' agendas. Our hope is to give voters the knowledge they need to mark ballots without any distorted sense of reality. Below is an example of that work — a months-long analysis of an all-encompassing effort to reshape the American bureacracy following the 2024 U.S. presidential election. If you'd like to support this type of journalism,  we'd love your help .   —  Jessica Lee ,  senior assignments editor,  snopes.com

As the U.S. 2024 presidential election nears, U.S. President Joe Biden's reelection campaign has been sending foreboding emails to supporters, invoking "Trump's Project 2025" to tap into anxieties over another four years with Donald Trump in the White House and to raise campaign money.

According to some of the emails, "Project 2025" calls for proposals that would separate "mothers away from their children," a reference to border policies during Trump's administration, or result in "higher housing costs and rampant discrimination."

The Biden campaign is not alone in its concern over the policy initiative. Critics including legal experts and former government employees have described Project 2025 as a precursor to authoritarianism — albeit a difficult one to implement — and a wave of social media  posts  are expressing  fear over the initiative, calling it a " fascist " and " extremist " plan for Trump to " reshape America." Numerous reports have also called this conservative effort to reshape the government unprecedented in its scale. 

But what exactly is Project 2025? Are the messages from critics rooted in fact or fear-mongering? What should people know about the alleged policy plan? Over the past year, Snopes has received a flood of inquiries from readers asking if Project 2025 was real and what it entails, and if American politicians plan to implement it.

Under the leadership of the Heritage Foundation, a conservative think tank, Project 2025 is indeed a real, all-encompassing initiative to transform the American bureaucracy if, or when, a conservative president takes over the White House. Project leaders are hoping to put it into motion as early as November 2024 if voters elect former President Donald Trump. 

Politico once described the policy initiative as an effort to make a "MAGA" conservative government by reshaping how federal employees work, and the  creators themselves have framed it as a push to institutionalize " Trumpism " —  that is,  Trump's political agenda — at every level of federal government. On Truth Social, a Trump-owned social media platform, users have described it as a return to "constitutional" values.

In June 2024, House Democrats launched a task force to make plans for a potential future in which Project 2025's recommendations could become reality.

The growing interest in Project 2025 coincided with the progression of Trump's presidential campaign. A  June 2024  NPR/PBS News/Marist poll found the presidential race to be extremely tight, with Biden and Trump almost tied, echoing a months-long trend of national surveys. ( Historically , polls at this stage of campaigns are not indicative of actual election outcomes.)

Leaders and supporters of the initiative declined to be interviewed for this story or did not respond to Snopes' inquiries.

What is Project 2025?

Project 2025 has four parts, according to its website : 

  • A roughly 1,000-page document titled " Mandate for Leadership 2025: The Conservative Promise ."  That report details supporters' proposals for federal departments, as well as their overall agenda for a conservative government.
  • A purported transition plan for federal departments. Project 2025 leaders say they have a 180-day transition plan for each federal agency to quickly adapt to a Trump presidency should he win in November. As of this writing, the contents of that plan were unknown.
  • A new database that aims to fill federal jobs with conservative voices. Spencer Chretien, associate director of Project 2025, once called the online system to screen potential new hires the " conservative LinkedIn ." It's currently active on the Project's website.
  • A new system to train potential political appointees . Called the " Presidential Administration Academy ," the system aims to teach skills for "advancing conservative ideas" as soon as new hires join the administration. The lessons touch on everything from budget-making to media relations and currently consist of 30- to 90-minute online sessions. Project 2025 leaders say they will host in-person sessions as the election nears. 

There's reportedly another facet to Project 2025 that's not detailed on its website: an effort to draft executive orders for the new president. According to a November 2023 report by The Washington Post that cites anonymous sources, Jeffrey Clark (a former Trump official who sought to use the Justice Department to help Trump's efforts to overturn 2020 election results) is leading that work, and the alleged draft executive orders involve the Insurrection Act — a law last updated in 1871 that allows the president to deploy the military for domestic law enforcement. Speaking to the Post, a Heritage spokesperson denied that accusation. (We were unable to independently corroborate The Washington Post's reporting due to its anonymous sourcing and our unsuccessful attempts to interview members of The Heritage Foundation.)

While many of Project 2025's proposals simply need the president's executive order to become reality, others would need Congressional approval, even as the Project seeks to expand presidential authority. In other words, lawmakers would need to write and approve legislation that details the changes to the government's existing structure, or establishes new systems. Come November, voters will choose who will fill  435 seats in the Republican-led House and 34 positions  in the Senate.

Key Points of The Roughly 1,000-Page Document

Speaking to Politico , Russell Vought, who served as the director of the Office of Management and Budget under Trump and is now a leading adviser for Project 2025, once described the effort as "more systematic than it is just about Trump," adding, "We have to be thinking mechanically about how to take these institutions over" in reference to federal departments.

Project 2025's document lays out in great detail how supporters want to do that. As of early June 2024, about 855,000 people had downloaded the document, The New York Times reported . 

Among its numerous recommendations, it calls for the following (in no particular order):

  • Changing how the FBI operates. According to the plan, the agency is "completely out of control," and the next conservative administration should restore its reputation by stopping investigations that are supposedly "unlawful or contrary to the national interest." Also, the document calls for legislation that would eliminate term limits for the FBI's director and require that person to answer to the president. 
  • Eliminating the Department of Education. The plan explicitly proposes, "Federal education policy should be limited and, ultimately, the federal Department of Education should be eliminated." The report also calls for bans on so-called " critical race theory" (CRT) and "gender ideology" lessons in public schools, asking for legislation that would require educators who share such material to register as sex offenders and be imprisoned. 
  • Defunding the Department of Justice. Additionally, the document proposes prosecuting federal election-related charges as criminal, not civil, cases. Otherwise, the document says, "[Voter] registration fraud and unlawful ballot correction will remain federal election offenses that are never appropriately investigated and prosecuted." 
  • Reversing Biden-era policies attempting to reduce climate change. The document's authors call for increasing the country's reliance on fossil fuels and withdrawing from efforts to address the climate crisis — such as "offices, programs, and directives designed to advance the Paris Climate Agreement ." 
  • Stopping cybersecurity efforts to combat mis- and disinformation. The document recommends the Cybersecurity and Infrastructure Security Agency to stop its efforts to curtail online propaganda campaigns, arguing the federal government should not make judgment calls on what's true and what isn't.
  • Changing immigration policies. Authors want the federal government to deprioritize DACA (Deferred Action for Childhood Arrivals), the program that temporarily delays the deportation of immigrants without documentation who came to the U.S. as children; phase out temporary work-visa programs that allow seasonal employers to hire foreign workers; impose financial punishments on so-called "sanctuary cities" that do not follow federal immigration laws, and divert tax dollars toward security at America's border with Mexico. (While the Biden campaign claims Project 2025 calls for "ripping mothers away from their children" at the border, there's no explicit mention of separating families. Rather, it calls for stronger enforcement of laws governing the detainment of immigrants with criminal records and restricting an existing program that tracks people in deportation proceedings instead of incarcerating them. In some cases, those changes could possibly play a role in border control agents detaining a parent while their child continues with immigration proceedings.)
  • Restricting access to abortion. The plan wants the Centers for Disease Control and Prevention (CDC) to stop promoting abortion as health care. Additionally, Project 2025 recommends the Federal Drug Administration (FDA) to stop promoting, and approving, requests for manufacturing abortion pills. "Alternative options to abortion, especially adoption, should receive federal and state support," the document states.
  • Removing LGBTQ+ protections. The plan calls for abolishing the Gender Policy Council , a Biden-created department within the White House that aims to "advance equity in government policy for those who face discrimination." Also, the proposal wants the federal government to remove terms such as "sexual orientation" and "gender identity" from records and policies, as well as rescind policies that prohibit discrimination on the basis of "sexual orientation, gender identity, transgender status, and sex characteristics."
  • Cutting ties completely with China. For instance, the document advocates for restricting people's access to TikTok because of its China-based parent company; prohibiting Confucius Institutes, cultural institutions at colleges and universities funded by the Chinese government, and blocking other Chinese entities from partnering with U.S. companies. 
  • Reversing protections against discrimination in housing. The Biden campaign emails reference a portion of the document that calls for repealing a decades-old policy—strengthened under Biden—that attempts to prevent discrimination and reduce racial disparities in housing. Project 2025 also recommends making it easier to sell off homes used for public housing — a benefit to real estate developers — but result in fewer cheap housing options for poor and low-income families. 

Here's a PDF of the full report :

(www.project2025.org)

Changing Federal Job Classifications 

To execute the above-listed objectives, the roughly 1,000-page document calls for a federal government operated by political appointees equipped to "carry out the President's desires." 

Put another way, Roberts, president of the Heritage Foundation, said in a July 2023 interview with The New York Times that Project 2025 leaders want to dismantle independent federal agencies that do not answer to the president. Then, they want to fill positions with people who subscribe to conservative politics — including jobs that are currently merit-based hires, not politically appointed.

Under the current system, the federal government's administrative sector is made up of two employee groups: political appointees and career civil servants. When a new administration takes over the Oval Office, it selects similarly minded people to fill high-ranking positions (political appointees), and those people leave the jobs when a new president takes over. According to the Brookings Institution , a public policy think tank, around 4,000 political appointees run the executive branch.

Meanwhile, tens of thousands of positions that run day-to-day operations are hired through a merit-based system — that is, a hiring process that is designed to prioritize applicants' specialized expertise or experience , not their personal beliefs or affiliations. Those people are career civil servants. 

Project 2025 proposes turning up to 50,000 career civil servant jobs into politically appointed positions. 

To do that, Project 2025 wants the president to reissue Schedule F, a Trump-era executive order that Biden rescinded when he became president. Generally speaking, the order would recategorize career civil servants into at-will employees, giving higher-level workers the ability to terminate employment for any reason without warning and fill those jobs with new people.

Additionally, Project 2025 recommends revamping the existing appeals process for employee dismissals, arguing the current system prevents managers from firing or hiring the right employees. 

The plan also proposes a freeze on hiring top-career civil service positions at the beginning of the administration. By doing so, the plan argues, the new administration will prevent today's administration's leaders (later on "outgoing" political appointees) from "burrowing-in"— that is, hiring left-leaning career bureaucrats across federal agencies for the purpose of undermining the next president. 

Keeping Track of Potential Employees' Opinions

In addition to expanding government leaders' abilities to hire and fire at will, Project 2025 calls for a new federal database to gather information on potential new hires. The database contains people's answers to questions on social issues , such as abortion and immigration, allowing for department leaders to easily fill job vacancies with applicants who lean conservative.

"Our current executive branch was conceived of by liberals for the purpose of promulgating liberal policies," John  McEntee , who is leading Project 2025's personnel database project, told The New York Times in mid-2023, citing then-U.S. President Franklin Delano Roosevelt's (who was a Democrat) 1930s New Deal as the last major reorientation of the government. "There is no way to make the existing structure function in a conservative manner. It's not enough to get the personnel right. What's necessary is a complete system overhaul." 

By submitting resumes and answering questionnaires , applicants sign up to be vetted by Project 2025 leaders. According to the questionnaire , participants answer whether they "agree" or "disagree" with statements such as, "Life has a right to legal protection from conception to natural death," and "The U.S. should increase legal immigration."

If the participants pass that screening, Project 2025 intends to recommend them to department leaders for hiring. (We are unable to determine what would happen with applicants' data if Trump does not win the 2024 election, or if his potential administration does not want to use it.)

Project 2025 leaders partnered with technology company Oracle to set up the system, according to The New York Times . Several thousand potential recruits had applied, as of April 2023. 

Former presidents have established similar systems, including Barack Obama, according to Kevin Kosar, a senior fellow at the American Enterprise Institute, a center-right public policy think tank. "They [The Obama administration] created a massive online jobs bank , where you could apply."

Also, during Obama's first term (January 2009 - January 2017), his administration required extensive vetting of applicants for high-ranking, politically appointed positions. Like Project 2025's program, that process included a questionnaire. That form asked participants to elaborate on past public statements, social media posts and potential conflicts of interests, as well as share things about their personal lives , like whether they own guns. (We found no evidence of the Obama administration circulating a similar questionnaire during his second term.)

Asked about that Obama-era questionnaire, a Biden aide said it was not comparable to Project 2025's system. The latter was a "loyalty test" to Trump, the aide said, while Obama's survey was more of a background check.

Trump Hasn't Publicly Endorsed Project 2025

Many former Trump administration members and current allies are working on the initiative. 

For example, the Center for Renewing America (CRA) — a think tank that formed in 2021 with ties to Trump through its founder, Russell Vought — is a "coalition partner." Vought was the director of the Office of Management and Budget when Trump was president. Should Project 2025 be a part of the next presidential administration, Vought will be in charge of implementing  its proposals, according to Politico. (In November 2023, The Washington Post reported he was in regular contact with Trump and could be a candidate for a high-ranking position in his potential future administration.) Also, Vought is policy director for the 2024 Republican National Convention's Platform Committee.

Reportedly , some people affiliated with Project 2025 are assisting Trump's reelection campaign behind the scenes.

how to write essay for ap lang

(The groups that conceptualized, or are currently pushing, Project 2025 include a number of former Trump administration members and current allies.)

However, in terms of public-facing actions, Trump hasn't officially connected himself to the initiative. In speeches at campaign rallies and interviews, he hasn't mentioned Project 2025, and, on July 5, 2024 , he attempted to publicly distance himself by posting on Truth Social (his social media site):

I know nothing about Project 2025. I have no idea who is behind it. I disagree with some of the things they're saying and some of the things they're saying are absolutely ridiculous and abysmal. Anything they do, I wish them luck, but I have nothing to do with them.

Trump's campaign is at the very least aware of the initiative. Campaign officials once told Politico Project 2025's goals to restructure government, which are outlined in a publicly available document , indeed align with Trump's campaign promises.

But in a November 2023 statement, the Trump campaign said: "The efforts by various non-profit groups are certainly appreciated and can be enormously helpful. However, none of these groups or individuals speak for President Trump or his campaign." Without naming Project 2025, they said all policy statements from "external allies" are just "recommendations."

Concurrently, in an interview with the conservative outlet The Daily Wire , a Project 2025 representative said the Trump campaign and Project are separate "for now."  McEntee , a former Trump staffer and leader of Project 2025's personnel database project, said : 

I think the candidate and the campaign need to keep their eye on the ball. They need to be totally focused on winning. We're totally focused on what happens after [...] Obviously, there will need to be coordination and the president and his team will announce an official transition this summer, and we're gonna integrate a lot of our work with them. 

That said, given overlap between Project 2025's proposals and the Trump campaign's agenda , political analysts and the Biden campaign believe the coalition's effort is a good indication of Trump's vision for a second term. Among the similarities are proposals to change how the administration fills tens of thousands of government jobs and overhaul  the DOJ. According to The Heritage Foundation's own reporting, Trump adopted and seriously considered about two-thirds of the organization's policy prescriptions in 2018, for example.

In an interview with Snopes, James Singer, a Biden campaign spokesperson, said:

Project 2025 is the extreme policy and personnel playbook for Trump's second term that should scare the hell out of any American voter. The Trump team's pathetic denials fall flat when Project 2025 staff and leadership are saying they are connected to the Trump team, leading the RNC policy platform and part of Trump's debate prep, campaign, and inner circle.

But the extent to which Project 2025 leaders and Trump campaign officials are communicating is unclear. According to Kosar, at the American Enterprise Institute, no one outside of the two circles knows how closely they're working together. "[What] is the level of coordination? We have no idea." 

From the view of Cecilia Esterline, an immigration research analyst at the Niskanen Center, a think tank  with libertarian-right roots, Project 2025 is a good indicator of Trump's plans for a potential second term. "Given the people involved putting their names on this and the author portions of this report, and the success of [past] implementation, it's a good indicator of where Trump is at."

The Forces Behind Project 2025

Heritage Foundation President Kevin Roberts launched Project 2025 in April 2022, a few months before Trump officially announced his reelection campaign.

Since then, the number of groups backing the initiative has grown. As of now, Project 2025's advisory board and so-called "coalition partners" include: the Conservative Partnership Institute (CPI), a nonprofit that aims to connect conservative applicants to congressional jobs and is led by Trump's former chief of staff, Mark Meadows; Turning Point USA, a far-right student advocacy group that is led by Charlie Kirk; America First Legal , a legal advocacy group that supports conservative-backed lawsuits and is led by Trump stalwart Stephen Miller. (According to a June 2024 Politico report, Miller was part of private meetings with Trump to help him prepare for upcoming televised debates against Biden.) 

Furthermore, in May 2024, Reuters interviewed what the news outlet described as unnamed Trump allies working on a plan to restructure the Department of Justice (DOJ) and fill currently nonpartisan jobs there with people who identify as conservatives. While the allies group wasn't named, Reuters reported it was tied to Project 2025. 

Lastly, many authors of the roughly 1,000-page document outlining Project 2025's policy proposals have connections to Trump. They include Ben Carson , William Perry Pendley , Jonathan Berry , Diana Furchtgott-Roth , Rick Dearborn , Adam Candeub , Ken Cuccinelli , Mandy Gunasekara , Dennis Dean Kirk , Gene Hamilton , Christopher Miller , Bernard L. McNamee , Mora Namdar , Peter Navarro , Roger Severino , Paul Dans , Kevin Roberts , among others. 

These Types of Pre-Election Efforts Aren't Uncommon

In the months or years before U.S. presidential elections, it's routine for nonprofit research groups to prepare plans for a potential presidential transition, according to Landon Storrs, a political history professor at the University of Iowa. 

And, according to Kosar, numerous think tanks want Trump's ear as he plans his potential return to the White House. "Whenever there is a new executive coming into the White House, [many] groups are trying to get in there."

According to the Heritage Foundation's website , the organization mostly operates on individual donations and does not take money from the government. However, how exactly it divvies up its money for Project 2025 was unclear. The New York Times reported Project 2025 was a $22 million operation.

Project 2025 authors built their proposals on an idea popular during former President Ronald Reagan's time: the "unitary executive theory." That's the belief that Article II of the U.S. Constitution gives the president complete power over the federal bureaucracy and all levels of government report to him. 

In 1980, the Heritage Foundation developed similar policy prescriptions for Reagan, who was a presidential candidate at the time. Some of the organization's recommendations aligned with Reagan's campaign promises , and, when he later assumed office, he put the ideas to action. Heritage once described its effort as putting "the conservative movement and Reagan on the same page."

However, according to Politico , the present-day initiative by the Heritage Foundation was more "ambitious" than any other such proposal. The New York Times  said Project 2025 was operating at "a scale never attempted before in conservative politics." Its efforts are a contrast to the 1930s Democrat-led New Deal under then-U.S. President Roosevelt, which gave the federal government an unprecedented role in social and economic affairs on the belief that it would get the country out of the Great Depression.

Critics' Logistical Concerns, Worries

If some of Project 2025's ideas turn into formal policy recommendations or laws, experts in government and history have concerns over how they could be implemented. Such drastic changes would come with big logistical hurdles and have a ripple effect on agencies overseeing day-to-day governance, several such experts said. 

For example, Project 2025's proposal to reclassify tens of thousands of federal workers' positions — that is, change career bureaucrats into jobs that can be politically appointed — would have widespread effects, according to Storrs, of the University of Iowa. She said:

When [Project 2025's] intention is to install officials based on their loyalty to the president rather than on their qualifications, [the result] is even more damaging to effective administration. [...] The President already has authority over who heads the agencies. But below them, people are simply trying to collect taxes, get social security checks out — there is a lot that shouldn't be disrupted.

Kosar, of the American Enterprise Institute, expressed concern over skills required for jobs that aren't currently appointed. "These positions have a serious degree of expertise attached. You can't just plug in a private sector businessman into the department of transportation. It's going to be a challenge to match the people and the competencies and the expertise." 

Esterline, the Niskanen Center analyst, said with presidential administrations changing every four to eight years, government agencies rely on the expertise of continually employed civil servants — employees with institutional knowledge — to make the transitions as smooth as possible. "[If] we suddenly disrupt that balance of political appointees to civil servants, it will be a much rougher transition." 

Among other aspects of Project 2025, Esterline is attempting to raise the alarm on its prescriptions for specific regulatory changes. "[Project 2025] is a meticulous outline of how they will crumple the system simultaneously through minute changes."

Meanwhile, some former government officials are particularly concerned about the initiative's plans for the DOJ and FBI. For instance, in an interview for The Guardian , Michael Bromwich, a former DOJ inspector general, said the proposals to turn the departments into "instruments" to fulfill Trump's political agenda "should send shivers down the spine of anyone who cares about the rule of law."

Overall, critics including legal experts and former government employees have zeroed in on Project 2025's goal to give the executive branch more power, describing it as a precursor to authoritarianism.

However, the initiative's push to increase executive power may be part of a deeper trend in American politics, Peter Strauss, a professor at Columbia Law School, said in a  lecture  on Faculti, a research video platform. He said momentum to increase executive authority has been steadily increasing over many presidential administrations: 

We have seen in the United States a steadily expanding presidential claim of authority to control not only tenure but also ordinary acts of government. This has been happening at least since the presidency of Ronald Reagan and it reached a peak with President Trump and his first term, and he's promised that he's going back there. 

Our Reporting

For this report, we repeatedly tried to interview representatives of the Heritage Foundation — the conservative think tank that conceptualized Project 2025 — as well as the Trump campaign and other supporters of the effort. All either declined to be interviewed or did not respond to our inquiries. 

For example, we reached out to dozens of groups on Project 2025's advisory board — a collection of groups under the Heritage Foundation's oversight that have co-signed the effort, given feedback on its proposals or promoted it to government officials. The groups include Center for Renewing America , Turning Point USA , The American Conservative , and  American Cornerstone Institute . We asked the organizations about the nature of their involvement in the initiative, proposals they support, and more. As of this writing, none has responded.

After we initially reached out to the Heritage Foundation for this story, a spokesperson responded asking for more specifics on our reporting. We responded with key points, including requests to comment on project leaders' communication with former U.S. President Donald Trump, concerns from legal experts about the initiative's proposed changes and general criticism. The Heritage Foundation did not respond to that message. Later, after informing the organization of our writing deadline, a spokesperson said no one was available.

"A Questionnaire for Applicants to the Next Conservative White House." The New York Times, 1 Dec. 2023. NYTimes.com, https://www.nytimes.com/interactive/2023/12/01/us/politics/project2025-heritage-foundation-administration-application-questionnaire.html. Accessed 20 June 2024.

"Affirmatively Furthering Fair Housing." HUD.Gov / U.S. Department of Housing and Urban Development (HUD), https://www.hud.gov/AFFH. Accessed 20 June 2024.

"Agenda47: Firing the Radical Marxist Prosecutors Destroying America." Donald J. Trump For President 2024. https://www.donaldjtrump.com/agenda47/agenda47-firing-the-radical-marxist-prosecutors-destroying-america. Accessed 20 June 2024.

"Agenda47: President Trump's Plan to Dismantle the Deep State and Return Power to the American People." Donald J. Trump For President 2024. https://www.donaldjtrump.com/agenda47/agenda47-president-trumps-plan-to-dismantle-the-deep-state-and-return-power-to-the-american-people. Accessed 20 June 2024.

Allen, Mike. "Scoop: Biden Campaign's Plan of Attack." Axios, 6 Mar. 2024, https://www.axios.com/2024/03/06/biden-trump-project-2025. Accessed 20 June 2024.

"American Cornerstone: A Much-Needed Endeavor." RealClearPolitics, Feb. 3, 2021. https://www.realclearpolitics.com/articles/2021/02/03/american_cornerstone_a_much-needed_endeavor_145170.html. Accessed 20 June 2024.

Arnsdorf, Isaac, et al. "Trump and Allies Plot Revenge, Justice Department Control in a Second Term." Washington Post, 6 Nov. 2023. https://www.washingtonpost.com/politics/2023/11/05/trump-revenge-second-term/. Accessed 20 June 2024.

Ball, Molly. "The Most Interesting Think Tank In American Politics." TIME, 7 Mar. 2023, https://time.com/6258610/niskanen-center-bipartisanship-think-tank-politics/. Accessed 20 June 2024.

Brugger, Kelsey. "Inside Heritage's Plans for a 2025 GOP Administration." E&E News by POLITICO, 22 Feb. 2023, https://www.eenews.net/articles/inside-heritages-plans-for-a-2025-gop-administration/. Accessed 20 June 2024.

Bump, Philip. "Analysis | A New Fox Poll Has Biden up. Fox's Prime-Time Hosts Didn't Mention It." Washington Post, 20 June 2024. www.washingtonpost.com, https://www.washingtonpost.com/politics/2024/06/20/new-fox-poll-shows-biden-up-their-primetime-hosts-didnt-mention-it/. Accessed 20 June 2024.

Calmes, Jackie. "For a Washington Job, Be Prepared to Tell All." The New York Times, 13 Nov. 2008. https://www.nytimes.com/2008/11/13/us/politics/13apply.html. Accessed 20 June 2024.

Chretien, Spencer. "Project 2025." The Heritage Foundation, https://www.heritage.org/conservatism/commentary/project-2025. Accessed 20 June 2024.

Cohn, Nate. "Trump Leads in 5 Key States, as Young and Nonwhite Voters Express Discontent With Biden." The New York Times, 13 May 2024. https://www.nytimes.com/2024/05/13/us/politics/biden-trump-battleground-poll.html. Accessed 20 June 2024.

"Conservatives Aim to Restructure U.S. Government and Replace It with Trump's Vision." PBS News, 29 Aug. 2023, https://www.pbs.org/newshour/politics/conservatives-aim-to-restructure-u-s-government-and-replace-it-with-trumps-vision. Accessed 20 June 2024.

Control of the Senate Is at Stake in 2024. Here Are the Top Races to Watch. https://ny1.com/nyc/all-boroughs/news/2024/04/11/top-u-s--senate-races-2024. Accessed 21 June 2024.

Ecarma, Caleb. "Donor Infighting and Dwindling Subscribers: The American Conservative May Be on Its Last Legs." Vanity Fair, 9 May 2023, https://www.vanityfair.com/news/2023/05/the-american-conservative-may-be-on-its-last-legs. Accessed 20 June 2024.

Esterline, Cecilia. "Project 2025: Unveiling the Far Right's Plan to Demolish Immigration in a Second Trump Term - Niskanen Center." Niskanen Center - Improving Policy, Advancing Moderation, 20 Feb. 2024, https://www.niskanencenter.org/project-2025-unveiling-the-far-rights-plan-to-demolish-immigration-in-a-second-trump-term/. Accessed 20 June 2024.

Esterline, Cecilia. Phone Interview.

"Excepted Service." Ballotpedia, https://ballotpedia.org/Excepted_service. Accessed 21 June 2024.

"Financial Information." The Heritage Foundation, https://www.heritage.org/financial. Accessed 20 June 2024.

"Former PPO Director John McEntee Joins Project 2025; Personnel Database Launches." The Heritage Foundation, https://www.heritage.org/press/former-ppo-director-john-mcentee-joins-project-2025-personnel-database-launches. Accessed 20 June 2024.

Garcia-Navarro, Lulu. "Inside the Heritage Foundation's Plans for 'Institutionalizing Trumpism.'" The New York Times, 21 Jan. 2024. NYTimes.com, https://www.nytimes.com/2024/01/21/magazine/heritage-foundation-kevin-roberts.html. Accessed 21 June 2024.

"Gender Policy Council." The White House, https://www.whitehouse.gov/gpc/. Accessed 20 June 2024.

Gross, Terry. "How the Trump White House Misled the World about Its Family Separation Policy." NPR, 11 Aug. 2022. NPR, https://www.npr.org/2022/08/11/1116917364/how-the-trump-white-house-misled-the-world-about-its-family-separation-policy. Accessed 21 June 2024.

Gupta, Alisha Haridasani. "Biden Signs Executive Order to Advance Gender Equity." The New York Times, 8 Mar. 2021. NYTimes.com, https://www.nytimes.com/2021/03/08/us/politics/gender-policy-council-biden.html. Accessed 21 June 2024.

Hirsch, Lauren, et al. "Companies Counter Pushback on Price Increases With Promotions." The New York Times, 1 June 2024. NYTimes.com, https://www.nytimes.com/2024/06/01/business/dealbook/companies-counter-pushback-on-price-increases-with-promotions.html. Accessed 21 June 2024.

Hirsh , Michael. "Inside the Next Republican Revolution." Politico, 19 Sept. 2023, www.politico.com/news/magazine/2023/09/19/project-2025-trump-reagan-00115811. Accessed 21 June 2024.

"House Democrats Step up to Try to Stop Project 2025 Plans for a Trump White House." AP News, 11 June 2024, https://apnews.com/article/project-2025-trump-biden-election-congress-6899a1167a4522b1c8be371f7abe7ee9. Accessed 21 June 2024.

"How Trump Secured the Delegates for the 2024 Republican Presidential Nomination." AP News, 12 Mar. 2024, https://apnews.com/article/trump-republicans-nomination-delegates-61801649ea454f730bd193ca04ab739f. Accessed 21 June 2024.

Isenstadt, Alex, and Meredith McGraw. "Trump Ramps up Debate Prep: 'Policy Discussions' Instead of Mock Debate." Politico, 17 June 2024, https://www.politico.com/news/2024/06/17/donald-trump-debate-prep-00163749. Accessed 21 June 2024.

Knefel, John. "The Center for Renewing America Has a Plan to Unleash the FBI on Its Political Enemies." Media Matters for America, 4 Jan. 2023, https://www.mediamatters.org/fox-news/center-renewing-america-has-plan-unleash-fbi-its-political-enemies. Accessed 21 June 2024.

Kosar, Kevin. Phone Interview.

Lawton, Sophie, et al. "A Guide to Project 2025, the Extreme Right-Wing Agenda for the next Republican Administration." Media Matters for America, 20 Mar. 2024, https://www.mediamatters.org/heritage-foundation/guide-project-2025-extreme-right-wing-agenda-next-republican-administration. Accessed 21 June 2024.

Lewis, Neil A. "300,000 Apply for 3,300 Obama Jobs." The New York Times, 6 Dec. 2008. NYTimes.com, https://www.nytimes.com/2008/12/06/us/politics/06seek.html. Accessed 21 June 2024.

Liles, Jordan and Taija PerryCook. "Trump Convicted on All 34 Counts of Falsifying Records in Hush-Money Case." Snopes, 30 May 2024, https://www.snopes.com//news/2024/05/30/donald-trump-verdict/. Accessed 21 June 2024.

"Mandate for Leadership: The Conservative Promise." The Heritage Foundation, 2023, thf_media.s3.amazonaws.com/project2025/2025_MandateForLeadership_FULL.pdf. Accessed 21 June 2024.

"MANDATE II." Washington Post, 26 Feb. 2024. www.washingtonpost.com, https://www.washingtonpost.com/archive/politics/1984/11/22/mandate-ii/ee2ede6b-86c0-4180-8e94-3d4a533a9b72/. Accessed 21 June 2024.

"Mark Meadows." Conservative Partnership Institute, https://www.cpi.org/staff/mark-meadows/. Accessed 20 June 2024.

Moore, Elena. "Biden and Trump Will Debate on Thursday. Here's What You Need to Know." NPR, 24 June 2024. NPR, https://www.npr.org/2024/06/24/nx-s1-5013579/trump-biden-presidential-debate-when. Accessed 25 June 2024.

North, Anna. "Trump May Sound Moderate on Abortion. The Groups Setting His Agenda Definitely Aren't." Vox, 8 Apr. 2024, https://www.vox.com/policy/24122099/trump-second-term-project-2025-christian-nationalists. Accessed 21 June 2024.

"Obama Moves into Hiring Mode." BBC, 15 Nov. 2008. http://news.bbc.co.uk/2/hi/americas/us_elections_2008/7730808.stm. Accessed 21 June 2024.

"Overseer or 'The Decider'? The American President in Administrative Law." Faculti, 5 Mar. 2024, https://faculti.net/overseer-or-the-decider/. Accessed 21 June 2024.

Palma, Bethania. "What Is Critical Race Theory and Why Are Some People So Mad at It?" Snopes, 27 May 2021, https://www.snopes.com//news/2021/05/27/what-is-critical-race-theory/. Accessed 21 June 2024.

"President Franklin Delano Roosevelt and the New Deal | Great Depression and World War II, 1929-1945." Library of Congress. https://www.loc.gov/classroom-materials/united-states-history-primary-source-timeline/great-depression-and-world-war-ii-1929-1945/franklin-delano-roosevelt-and-the-new-deal/. Accessed 27 June 2024.

"Project 2025 Reaches 100 Coalition Partners, Continues to Grow in Preparation for Next President." The Heritage Foundation, https://www.heritage.org/press/project-2025-reaches-100-coalition-partners-continues-grow-preparation-next-president. Accessed 20 June 2024.

"Project 2025 Advisor Says the Initiative Will 'Integrate a Lot of Our Work' with the Trump Campaign Later This Year." Media Matters for America, 22 Apr. 2024, https://www.mediamatters.org/project-2025/project-2025-advisor-says-initiative-will-integrate-lot-our-work-trump-campaign-later. Accessed 21 June 2024.

"RNC, TRUMP CAMPAIGN ANNOUNCE LEADERSHIP FOR 2024 REPUBLICAN NATIONAL CONVENTION'S PLATFORM COMMITTEE." GOP.Com, 15 May 2024, gop.com/press-release/rnc-trump-campaign-announce-leadership-for-2024-republican-national-conventions-platform-committee/. Accessed 21 June 2024.

"Senior Executive Service." Ballotpedia, https://ballotpedia.org/Senior_Executive_Service. Accessed 21 June 2024.

Shortis, Emma. "Friday Essay: Project 2025, the Policy Substance behind Trump's Showmanship, Reveals a Radical Plan to Reshape the World." The Conversation, 25 Apr. 2024, http://theconversation.com/friday-essay-project-2025-the-policy-substance-behind-trumps-showmanship-reveals-a-radical-plan-to-reshape-the-world-227161. Accessed 21 June 2024.

Singer, James. Biden Campaign. Email Interview.

Slattery, Gram, et al. "Donald Trump Wants to Control the Justice Department and FBI. His Allies Have a Plan." Reuters, 29 May 2024. www.reuters.com, https://www.reuters.com/world/us/donald-trump-wants-control-justice-department-fbi-his-allies-have-plan-2024-05-17/. Accessed 21 June 2024.

Stone, Peter. "A Far-Right US Youth Group Is Ramping up Its Movement to Back Election Deniers." The Guardian, 2 Mar. 2024. The Guardian, https://www.theguardian.com/us-news/2024/mar/02/far-right-youth-group-turning-point-charlie-kirk. Accessed 21 June 2024.

Stone, Peter. "Money and Misinformation: How Turning Point USA Became a Formidable pro-Trump Force." The Guardian, 23 Oct. 2021. The Guardian, https://www.theguardian.com/us-news/2021/oct/23/turning-point-rightwing-youth-group-critics-tactics. Accessed 21 June 2024.

Stone, Peter. "'Openly Authoritarian Campaign': Trump's Threats of Revenge Fuel Alarm." The Guardian, 22 Nov. 2023. The Guardian, Nov. 22, 2023. https://www.theguardian.com/us-news/2023/nov/22/trump-revenge-game-plan-alarm. Accessed 21 June 2024.

Storrs, Landon. Phone Interview.

Stracqualursi, Veronica and Gabby Orr, Kristen Holmes. "Former President Donald Trump Announces a White House Bid for 2024." CNN, 16 Nov. 2022, https://www.cnn.com/2022/11/15/politics/trump-2024-presidential-bid/index.html. Accessed 21 June 2024.

Swan, Jonathan, et al. "Trump and Allies Forge Plans to Increase Presidential Power in 2025." The New York Times, 17 July 2023. NYTimes.com, https://www.nytimes.com/2023/07/17/us/politics/trump-plans-2025.html. Accessed 21 June 2024.

Swan, Jonathan, and Maggie Haberman. "Heritage Foundation Makes Plans to Staff Next G.O.P. Administration." The New York Times, 20 Apr. 2023. NYTimes.com, https://www.nytimes.com/2023/04/20/us/politics/republican-president-2024-heritage-foundation.html. Accessed 21 June 2024.

"The 2024 Executive Power Survey – Unitary Executive." The New York Times, 15 Sept. 2023. NYTimes.com, https://www.nytimes.com/interactive/2023/us/politics/unitary-executive-executive-power.html. Accessed 21 June 2024.

"The Paris Agreement." UNFCCC. https://unfccc.int/process-and-meetings/the-paris-agreement. Accessed 20 June 2024.

"The Principles of Trumpism." The Heritage Foundation, https://www.heritage.org/event/the-principles-trumpism. Accessed 26 June 2024.

"The Risks of Schedule F for Administrative Capacity and Government Accountability." Brookings, https://www.brookings.edu/articles/the-risks-of-schedule-f-for-administrative-capacity-and-government-accountability/. Accessed 21 June 2024.

"TikTok Content Creators Sue the US Government over Law That Could Ban the Popular Platform." AP News, 14 May 2024, https://apnews.com/article/tiktok-creators-sue-ban-bytedance-3324b0fee4473f6e05c5c51ae5ff6946. Accessed 21 June 2024.

"Tracking the Criminal and Civil Cases against Donald Trump." Associated Press, 17 Nov. 2023, https://apnews.com/projects/trump-investigations-civil-criminal-tracker. Accessed 20 June 2024.

"Trump Administration Embraces Heritage Foundation Policy Recommendations." The Heritage Foundation, https://www.heritage.org/impact/trump-administration-embraces-heritage-foundation-policy-recommendations. Accessed 20 June 2024.

Trump Wants to Be America's Landlord | The Nation. 14 June 2024, https://web.archive.org/web/20240614115404/https://www.thenation.com/article/society/housing-policy-hud-section-8-real-estate/. Accessed 20 June 2024.

Under a Second Trump Term, the DHS Will Be Even Harsher Than Before | The Nation. 5 June 2024, https://web.archive.org/web/20240605003444/https://www.thenation.com/article/society/project-2025-dhs-immigration/. Accessed 20 June 2024.

"United States Congress Elections, 2024." Ballotpedia, https://ballotpedia.org/United_States_Congress_elections,_2024. Accessed 21 June 2024.

"What You Need to Know about Jeffrey Clark's 2020 Election Charges." PBS News, 8 Sept. 2023, https://www.pbs.org/newshour/politics/what-you-need-to-know-about-jeffrey-clarks-2020-election-charges. Accessed 20 June 2024.

"Who We Are." The Center for Renewing America, https://americarenewing.com/about/. Accessed 20 June 2024.

July 5, 2024: This post was updated to include Trump's July 5, 2024, post on Truth Social.

By Nur Ibrahim

Nur Nasreen Ibrahim is a reporter with experience working in television, international news coverage, fact checking, and creative writing.

By Aleksandra Wrona

Aleksandra Wrona is a reporting fellow for Snopes, based in the Warsaw area.

Article Tags

IMAGES

  1. How To Write An AP Lang Argumentative Essay?

    how to write essay for ap lang

  2. How to Write the AP Lang Rhetorical Essay

    how to write essay for ap lang

  3. ⭐ Ap english language and composition rhetorical analysis essay sample

    how to write essay for ap lang

  4. How to Write a Synthesis Essay

    how to write essay for ap lang

  5. How To Write An Open Ended Essay Ap Lang

    how to write essay for ap lang

  6. How To Write An AP Lang Argumentative Essay?

    how to write essay for ap lang

VIDEO

  1. What is the difference between AP Lang and AP Lit?

  2. I Wrote an Ebook!

  3. CONQUER Rhetorical Analysis Commentary with THIS!

  4. Writing Rhetorical Analysis was HARD Until I Did THIS!

  5. How to Improve Your Writing Style

  6. AP LANG

COMMENTS

  1. How to Write the AP Lang Synthesis Essay + Example

    Step 5: Write your Essay. Use the remaining 30-35 minutes to write your essay. This should be relatively easy if you took the time to mark up the sources and have a detailed outline. Remember to add special consideration and emphasis to the commentary sections of the supporting arguments outlined in your thesis.

  2. How to Write the AP Lang Argument Essay (With Example)

    Her story "The Astronaut" won the 2018 Shirley Jackson Award for short fiction and received a "Distinguished Stories" mention in the 2019 Best American Short Stories anthology. Ap Lang Argumentative Essay - Expert advice on how to pen a winning essay + an AP Lang argument essay example to guide your writing.

  3. How to Write a Perfect Synthesis Essay for the AP Language Exam

    Paragraph 1: The prompt presents and briefly explains the topic that you'll be writing your synthesis essay about. That topic is the concept of eminent domain. Paragraph 2: The prompt presents a specific claim about the concept of eminent domain in this paragraph: Eminent domain is productive and beneficial.This paragraph instructs you to decide whether you want to defend, challenge, or ...

  4. How to Write the AP Lang Argument Essay + Examples

    2. Pick one side of the argument, but acknowledge the other side. When you write the essay, it's best if you pick one side of the debate and stick with it for the entire essay. All your evidence should be in support of that one side. However, in your introductory paragraph, as you introduce the debate, be sure to mention any merit the ...

  5. How to Write the AP Lang Rhetorical Essay

    Tips for Writing the AP Lang Rhetorical Essay. 1. Outline Your Essay Before Writing. One of the most important parts of the AP Lang essays is structuring your essay so that it makes sense to the reader. This is just as important as having good content. For this essay in particular, you'll want to read the passage first and write a brief ...

  6. How to Write the AP Lang Synthesis Essay with Example

    AP Lang Exam - Scoring. In the second part of the AP Lang exam, students can earn a possible 6 points on each essay. 1 point is earned for the development of a thesis. Up to 4 points can be earned for evidence and commentary. The final 1 point is earned for sophistication of thought. AP Lang Exam - Takeaways

  7. How to Ace the AP Language Rhetorical Analysis Essay

    In this video, I'll show you how to write the AP English Language rhetorical analysis essay (Q2) step by step using the actual 2017 prompt. Watch me annotate...

  8. Acing the AP® English Language and Composition Synthesis Essay

    The newest section of the AP® English Language and Composition Exam, the synthesis essay, is one of three essays you will be completing during the examination's 2-hour free-response period. However, you'll also have a 15-minute reading and planning period just for this essay, and if you use this time to plan effectively, you can't go wrong.

  9. Crafting an Impressive Argumentative Essay for AP Lang

    Writing an argumentative essay for AP Language and Composition requires a strategic approach to effectively convey your perspective. Here's a guide to crafting an impressive argumentative essay: 1. Understand the Prompt: - Tip: Carefully read and analyze the prompt. Identify the key elements, including the task you are asked to perform and any ...

  10. Writing a synthesis essay for AP Lang

    Sure, a synthesis essay is a type of essay that requires you to use multiple sources to create an argument. In an AP Lang synthesis essay, you'll typically be provided with the sources and will need to analyze them, identify the main ideas, and then connect those ideas to your central argument or thesis. Here's a step-by-step breakdown of how to write a strong synthesis essay for AP Lang: 1.

  11. How to Get a 6 on Argument FRQ in AP® English Language

    Pick an opinion and stick to it. Choose one side of the argument and one clear claim to support all the way through. Craft a thesis statement. Your thesis should be clear, concise, and introduce the content of your essay. Craft a chronological argument. Make an argument that builds on its prior points.

  12. AP English Language and Composition

    Section I: Multiple Choice. 45 Questions | 1 hour | 45% of Exam Score. Includes 5 sets of questions: 23-25 Reading questions that ask students to read and analyze nonfiction texts. 20-22 Writing questions that ask students to "read like a writer" and consider revisions to stimulus texts.

  13. How to Write the AP Lang Synthesis Essay + Essay Template

    The use of rhetorical devices AP Lang is also pretty important. So once you flesh out your essay a bit, spend some time trying to come up with the perfect wording. Step 5. Finalize. The first finished version of your essay is a draft. Don't be hasty to turn it in. Read over it a couple of times.

  14. AP Lang Exam Guide

    Format of the 2024 AP English Language and Composition exam. This year, all AP exams will cover all units and essay types. The 2024 AP English Language and Composition exam format will be: Section I: Multiple Choice - 45% of your score- - 45 questions in 1 hour. Section II: Free Response Section - 55% of your score- - 2 hours and 15 minutes for ...

  15. AP English Language and Composition: Sample Rhetorical Analysis and

    AP English Language and Composition: Sample Argument Question. The following paragraph is adapted from Mirror for Man, a book written by anthropologist Clyde Kluckhorn in the middle of the twentieth century. Read the passage carefully. Then, write an essay that examines the extent to which the author's characterization of the United States ...

  16. Synthesis Essay Materials

    The two synthesis essay questions below are examples of the question type that has been one of the three free-response questions on the AP English Language and Composition Exam as of the May 2007 exam. The synthesis question asks students to synthesize information from a variety of sources to inform their own discussion of a topic. Students are given a 15-minute reading period to accommodate ...

  17. Expert Guide to the AP Language and Composition Exam

    The AP English Language and Composition Multiple-Choice. The multiple-choice section tests you on two main areas. The first is how well you can read and understand nonfiction passages for their use of rhetorical devices and tools. The second is how well you can "think like a writer" and make revisions to texts in composition questions.

  18. How to Ace the AP Language Synthesis Essay

    In this video, I'll show you how to write the AP English Language synthesis essay (Q1) step by step using the actual 2017 prompt. This tutorial also includes...

  19. AP Lang Argument Essay: Evidence

    It is typically based on the knowledge and experience of the person offering it. Argument FRQ: An Argument FRQ (Free Response Question) is an essay question on the AP English Language exam that requires students to construct and defend an argument using evidence and rhetorical strategies. Audience: The audience refers to the intended recipients ...

  20. AP English Language and Composition Writing Study Skills

    In AP English, writing is taught as "process"—that is, thinking, planning, drafting the text, then reviewing, discussing, redrafting, editing, polishing, and finishing it. It's also important that AP students learn to write "on call" or "on demand." Learning to write critical or expository essays on call takes time and practice.

  21. Ultimate Guide to the AP English Language and Composition Exam

    AP Language and Composition Score Distribution, Average Score, and Passing Rate. In 2019, 54.3% of the students who took the AP English Language and Composition exam received a score of 3 or higher. Only 9.9% of students who took the exam achieved the top score of 5, and 14.5% of students who took the exam scored a 1.

  22. The Ultimate Guide to Acing the AP English Language and Composition

    The free-response section of the AP English Language and Composition exam requires students to write three essays that address three distinct tasks: synthesis, rhetorical analysis, and argument. Here are some tips on how to best tackle this section: Read the prompts carefully: Ensure you understand what each prompt asks you to do before writing.

  23. Guide for AP Lang Synthesis Essay : r/APStudents

    Then add a transition that can help your flow on your essay it can be very simple. After that, add your second piece of quote from a source to serve as evidence. (similarly and in addition are great transitions). Add commentary (at least 3 sentences). Repeat steps 3-6 for your second body paragraph. Add a conclusion if you have time.

  24. What to do about a 3 on AP Lang?

    You may consider a tutor then. Or summer class (it's late). You will need to write multiple essays for the colleges you seek to attend. Also you got a 3. So you did competent work. So I don't think, personally, that you can't write an essay. You wrote one and earned a decent score.

  25. Republicans move at Trump's behest to change how they will oppose ...

    MILWAUKEE (AP) — The Republican National Committee moved Monday to adopt a party platform that reflects former President Donald Trump's position opposing a federal abortion ban and ceding ...

  26. What's Project 2025? Unpacking the Pro-Trump Plan to Overhaul US

    In other words, lawmakers would need to write and approve legislation that details the changes to the government's existing structure, or establishes new systems.